pediatric hesi

Lakukan tugas rumah & ujian kamu dengan baik sekarang menggunakan Quizwiz!

In a clinic, the mother of an 8-month old asks the nurse what to feed her infant because she wants to stop breast feeding. The nurse recommends :

formula

describe feeding techniques for a child with cleft lip or palate

use lamb's nipple or prosthesis feed child upright, with frequent bubbling

Immunization

Cold doesn't stop it unless > 99 Use acetaminophen orally

Hesi hint

Mobiles: infant Puppets: toddler/preschool Cars: preschool Games: school age Themselves: adolescent

Hgb norms

Newborn: 14 to 24 g/dl Infant: 10 to 15 g/dl Child: 11 to 16 g/dl

what is compartment syndrome?

damage to nerves and vasculature of an extremity due to compression

Described the function of an osmotic diuretic

osmotic diuretics remove water form the CNS to reduce cerebral edema

what is the relationship between hypoglycemia and exercise?

during exercise, insulin uptake is increased and the risk for hypoglycemia occurs

identify food sources of vitamin A

liver, sweet potatoes, carrots, spinach, peaches and apricots

Cyanotic heart defects

Tetralogy of fallot: 4 defects Transposition of the great vessels: everything is criss crossed Truncus arterious : single great vessel instead of 2

When discussing discipline with the mother of a 4 yr old child, the nurse should include which guideline?

parental control should be consistent

What are the three classic signs of diabetes?

polydipsia polyphagia polyuria

A 6-month old male is at his well-child visit. The nurse weighs him, and his mom ask if his weight is normal for his age. The nurse's best response is?

"At 6 months his weight should be approximately twice his birth weight."

Following the administration of immunizations to a 6-month-old girl, the nurse provides the family with home care instructions. Which statement by the mother indicates that further teaching is needed?

"I will give her a baby aspirin every 4 hours as needed for fever." Rationale: Although fever may occur, non-aspirin-containing medications should be used because of the risk of Reye's syndrome

A bottle-fed infant, age 3 months, is brought to the ped. office for well child visit. At previous visit, nurse taught mom about nutritional needs. Which statement by the mother during the current visit indicates effective teaching?

"I'm giving my baby iron-fortified formula and a flouride supplement because our water isn't flouridated."

The nurse is taking the family history of a 2-year-old child with atopic dermatitis (eczema). Which statement by the mother is most important in formulating a plan of care for this child?

"My husband and our daughter are both lactose-intolerant." Rationale: Environmental exposure to allergens (milk) and a positive family history for milk allergies are important data in planning care of the child with atopic dermatitis because milk allergies can contribute to the child's outbreaks.

A 7-month-old male infant diagnosed with spastic cerebral palsy is seen by the nurse in the clinic. Which statement by the parent warrants immediate intervention by the nurse?

"My son often chokes while I am feeding him." Rationale: Airway obstruction is always a priority when caring for any client

The nurse is assessing a male adolescent client's knowledge of contraception. The teen states, "I have all the info I need." What is the best response by the nurse?

"Tell me what you know about birth control." Rationale: Teens often obtain information from peers, which may not be accurate. Knowing the source of the information may assist the nurse in evaluating the information that the teenager has regarding contraception

A woman whose first child died at 6 weeks of age because of sudden infant death syndrome (SIDS) is being discharged following the birth of her second child. The mother tells the nurse that she is fearful that this infant will also develop SIDS. Which response is best for the nurse to provide this woman?

"The fear of losing another child to SIDS is very realistic. Have you thought about what support you may need?" Rationale: The most effective way to provide emotional support is to acknowledge what clients may be feeling, be a sounding board for them so they can listen to themselves, and allow them to discover their own solutions

A 3yo female is hospitalized for an ASD repair. Her parents have decided to go home for a few hours to spend time with her siblings. The child asks when her mommy and daddy will be back. The nurse's best response is:

"Your mommy and daddy will be back after your nap."

A home care nurse is providing instructions to a child with cystic fibrosis about how to perform the "huff" maneuver; the child asks the nurse about the purpose of this type of breathing. The nurse makes which response to the child? a. "this type of breathing is used to mobilize secretions so that they can be easily coughed out" b. "this type of breathing prolongs inspiration time" c. "this type of breathing moves air out of the lower lungs" d. "this type of breathing moves air through the lungs"

"this type of breathing is used to mobilize secretions so that they can be easily coughed out"

A simple fracture is a fracture of the bone across its entire shaft with some possible displacement but without breaking the skin. A greenstick fracture is an incomplete fracture that occurs through only a part of the cross section of the bone; one side of the bone is fractured, and the other side is bent. A compound fracture, also called an open or a complex fracture, is one in which the skin or mucous membrane has been broken, and the wound extends to the depth of the fractured bone. A comminuted fracture is a complete fracture across the shaft of the bone with splintering of the bone fragments.

.

ephrotic syndrome is a kidney disorder characterized by massive proteinuria, hypoalbuminemia, edema, elevated serum lipids, anorexia, and pallor. The child gains weight.

.

the nurse practicioner is giving an inservice on infectious diseases and explains that the time interval between early manifestations of the disease incubation—how long takes to establish infection prodromal

...

Glomerulonephritis is a term that refers to a group of kidney disorders characterized by inflammatory injury in the glomerulus. In glomerulonephritis, activity is limited, and most children, because of fatigue, voluntarily restrict their activities during the active phase of the disease.

...

the nurse is concerned with prevention of communicable disease. Primary prevention results from which of following immunization early diagnosis strict isolation treatment of disease

...

Which development is necessary for toilet training readiness for a 2-year-old? Select all that apply. 1) Adequate neuromuscular development for sphincter control. 2) Appropriate chronological age. 3) Ability to communicate the need to use the toilet. 4) Desire to please the parent. 5) Ability to play with other 2-year-olds.

1, 3, 4. Readiness for toilet training is based on neurological, psychological, and physical developmental readiness. The nurse can introduce concepts of readiness for toilet training and encourage parents to look for adaptive and psychomotor signs such as the ability to walk well, balance, climb, sit in a chair, dress oneself, please the parent, and communicate awareness of the need to urinate or defecate.

The parents of a preschooler ask the nurse how to handle their child's temper tantrums. Which of the following should the nurse include in the teaching plan? Select all that apply. 1) Putting the child in "time-out." 2) Telling the child to go to his bedroom. 3) Ignoring the child. 4) Putting the child to bed. 5) Spanking the child. 6) Trying to reason with the child.

1, 3. Some parents find that putting the child in time-out until control is regained is very effective. Others find that ignoring the behaviors works just as well with their child.

A 10-month-old child with bronchiolitis is taken out of the 30% oxygen tent for breakfast because he refuses to eat unless in a high chair. During the feeding, the nurse notes that the child's respiratory rate has increased, he is becoming more irritable, and he is using accessory muscles to breathe. The first action of the nurse should be to: 1. Discontinue the feeding and place the child back in the tent. 2. Assess the pulse rate and respirations and notify the physician. 3. Perform postural drainage and then complete the feeding. 4. Suction the child's nose with a bulb syringe.

1.

A 7-month-old female infant is admitted to the hospital with a tentative diagnosis of Hirschsprung's disease. When obtaining the infant's initial health history from the parents, which of the following statements made by the mother would be most important? 1. She gets constipated often. 2. Sometimes she gets colds. 3. She spits up occasionally. 4. Her rectal temperature is 99.4

1.

A child is receiving methylprednisolone (Solu-Medrol) I.V. as treatment for a severe asthmatic attack. The nurse closely monitors the flow rate of the I.V. infusion to prevent the development of which of the following? 1. Hypertension. 2. Nausea. 3. Flushing of the skin. 4. Seizures.

1.

A community health nurse has taught a parent in the clinic about the ages that children receive immunizations and the reason why certain immunizations, such as the measles, mumps, rubella and polio vaccines, are given at different times. The nurse should judge the teaching as successful when she overhears this parent tell another parent: 2. My 6-month-old child will have to wait for the MMR vaccine. 2. My child has a cold and will have to wait 2 weeks to receive immunizations. 3. Children must wait 2 months between the MMR and polio vaccines. 4. Children receive their MMR vaccine and then have to wait 1 month for the tuberculin skin test.

1.

The nurse should explain that the most common cause for the unhappiness some children experience when first entering school is due to which of the following? 1. Feelings of insecurity. 2. Social isolation. 3. Emotional maladjustment. 4. Poor language development.

1.

When performing a physical assessment on an 18-month old child, which of the following would be best? 1. Have the mother hold the toddler on her lap. 2. Assess the ears and mouth first. 3. Carry out the assessment from head to toe. 4. Assess motor function by having the child run and walk.

1.

When teaching a group of parents of school-age children about growth and development, which of the following characteristics about children of this age should the nurse include? 1. Desire to carry a task to completion. 2. Ability to imagine possibilities. 3. Feeling that others are focused on them. 4. Ability to consider hypothetical risks and benefits.

1.

When the nurse asks a child suspected of being physically abused how his shoulder was hurt, he replies "it was my fault. I was bad." What would be the nurse's best response? 1. Perhaps it wasn't your fault. Can we talk about what happened? 2. Tell me what you did that made your father hurt you. 3. We'll make you better and we won't let your father do this to you again. 4. You'll have to behave better so this won't happen again.

1.

Which of the following information during a health history should the nurse correlate as consistent with the diagnosis of failure to thrive in an infant? 1. Fussiness during feedings. 2. Fear of strangers. 3. Being quiet when held. 4. Needing to be awakened for feedings.

1.

Which of the following should the nurse do next after noting that an 8-month-old child's posterior fontanel is slightly open? 1. Check the child's head circumference. 2. Document this as a normal finding. 3. Question the mother about the child's delivery. 4. Schedule an x-ray of the child's head.

1.

While attending a support group, the parents of a child with hemophilia become concerned because several of the families have had older children who have died from acquired immunodeficiency syndrome (AIDS). They ask the nurse how these children got the AIDS virus. The nurse bases the response on which of the following as the most likely route of transmission of AIDS to these children? 1. Contamination of the factor VIII replacement received during bleeding episodes. 2. Casual contact with a child testing positive for human immunodeficiency virus. 3. Use of a contaminated needle to obtain a blood sample for type and crossmatching. 4. Exposure in the waiting room to children with AIDS attending the same hematology clinic.

1.

Increased intracranial pressure is suspected in a 4-year-old child exhibiting a decreased level of consciousness. Which of the following assessment findings should also be of most concern to the nurse? 1. Blood pressure of 122/74. 2. Pulse of 86 beats/minute. 3. Respiratory rate of 24 breaths/minute. 4. Temperature of 100.2 F

1. A blood pressure of 122/74 is above the 95th percentile for a 4-year-old child. Increased blood pressure is a common sign of increased intracranial pressure.

A nurse is performing a Denver Developmental Screening Test (Denver II) on a 4-year old. The nurse determines that the test has resulted in a caution score when there are: 1. Failed or refused items intersected by the age line between the 25th and 75th percentiles. 2. A large number of refusals to the right of the age line. 3. More failures than passes along the age line. 4. Passed or failed items intersected by the age line in the 25th and 75th percentiles.

1. A caution score is given when there are failed or refused items intersected by the age line between the 25th and 75th percentiles.

A parent asks why it is recommended that the second dose of the measles, mumps, and rubella (MMR) vaccine be given at 4 to 6 years of age? The nurse should explain to the parent that the second dose is given at this age for what reason? 1) If the child reaches puberty and becomes pregnant when receiving the vaccine, the risks to the fetus are high. 2) The change of contracting the disease is much lower at this age. 3) The dangers associated with a strong reaction to the vaccine are increased at this age. 4) A serious complication from the vaccine is swelling of the joints.

1. After receiving the MMR vaccine, the person develops a mild form of the disease, stimulating the body to develop an immunity. Administration to a pregnant adolescent early in pregnancy puts the fetus at risk for deformity or spontaneous abortion.

The parents report that the child has a runny nose, fever, and cough and is irritable and constantly rubbing his ears. How should the nurse expect the child's tympanic membrane to appear? 1. Bulging and red. 2. Clear and inverted. 3. Pearly gray. 4. Scarred.

1. Based on the report of the child's signs and symptoms, the nurse should suspect otitis media.

A child with rheumatic fever has chorea. Which of the following actions should the nurse consider to be most important? 1. Explain to the child and family that the chorea will disappear over time. 2. Institute measures to keep the child in a warm environment. 3. Perform neurologic checks every 4 hours until the chorea subsides. 4. Encourage ambulation by giving aspirin 30 minutes before walking.

1. Because the clumsiness and uncontrolled actions can be upsetting to both the child and family, they need to understand that chorea associated with rheumatic fever is not permanent.

After teaching a group of parents about temper tantrums, the nurse knows the teaching has been effective when one of the parents states which of the following? 1) I will ignore the temper tantrum. 2) I should pick up the child during a tantrum. 3) I'll talk to my daughter during the tantrum. 4) I should put my child in time out.

1. Children who have temper tantrums should be ignored as longs as they are safe. They should not receive either positive or negative reinforcement to avoid perpetuating the behavior.

In the initial assessment, which sign should the nurse expect as typical of esophageal atresia and tracheoesophageal fistula? 1. Continuous drooling. 2. Diaphragmatic breathing. 3. Bloody emesis. 4. Large amounts of frothy meconium.

1. Esophageal atresia and tracheoesophageal fistula may occur together or separately. Esophageal atresia prevents the passage of swallowed mucus and saliva into the stomach.

Which of the following statements by an aolescent receiving gentamicin sulfate (Garamycin) should the nurse interpret as indicating drug toxicity? 1. I'm feeling dizzy. 2. I have no appetite. 3. I urinate a lot now. 4. I haven't moved my bowels in 3 days.

1. Gentamicin sulfate is a broad-spectrum aminoglycoside antibiotic that can cause nephrotoxicity and ototoxicity. Manifestations of ototoxicity include hearing problems and vestibular disturbances, such as dizziness.

A child with leukemia has petechiae; gums, lips, and nose that bleed easily; and bruising on various parts of her body. Which of the following laboratory test results should the nurse correlate with these findings? 1. Platelet count of 80. 2. Serum calcium level of 5 3. Fibrinogen level of 75. 4. Partial thromboplastin time of 38 seconds.

1. In leukemia, megakaryocytes, from which platelets are derived, are decreased. Normal counts range from 150 to 300.

When assessing the child with asthma for allergic rhinitis, which of the following should the nurse expect to find? 1. Nasal crease. 2. Abdominal pain. 3. Fever. 4. Mouth breathing.

1. In the child with asthma and allergic rhinitis, the allergic reaction to inhaled particles generally causes frequent nose rubbing, subsequently leading to a nasal crease.

The parents of a 9-month-old bring the infant to the clinic for a regular checkup. The infant has received no immunizations. Which of the following would be appropriate for the nurse to administer at this visit? 1) Diphtheria, tetanus, and acellular pertussis (DTaP); Haemophilus influenzae type B (Hib); inactivated poliomyelitis vaccine (IPV); and purified protein derivitive (PPD). 2) DTaP, Hib, oral polio vaccine (OPV), and measles, mumps, and rubella (MMR). 3) PPD, MMR, hepatitis B (hepB), and OPV 4) HepB, IPV, Hib, and varicella.

1. Infants that are delayed in receiving their immunizations or have not started their series by 9months of age begin with DTaP, Hib, IPV, and PPD.

A mother states that she thinks her 9-month-old "is developing slowly." When assessing the infant's development, the nurse is also concerned because the infant should be demonstrating which of the following characteristics? 1) Vocalizing single syllables. 2) Standing alone. 3) Building a tower of two cubes. 4) Drinking from a cup with little spilling.

1. Normally a 9-month-old infant should have been voicing single syllables since 6 months of age.

For a child receiving steroids in therapeutic doses over a long period, the nurse should: 1. Monitor the child's serum glucose level. 2. Decrease the child's ingestion of potassium-rich foods. 3. Give the drug on an empty stomach. 4. Monitor the child's temperature to asses for infection.

1. Steroid use tends to elevate glucose levels. The child should be monitored for increases.

A mother has heard that several children have been diagnosed with mononucleosis. She asks the nurse what precautions should be taken to prevent this from occurring in her child. Which of the following should the nurse advise the mother to do? 1) Take no particular precautionary measures. 2) Sterilize the child's eating utensils before they are reused. 3) Wash the child's linens separately in hot, soapy water. 4) Wear masks when providing direct personal care.

1. The cause of infectious mononucleosis is thought to be the Epstein-Barr virus. It is believed to be spread only by direct intimate contact.

The parents of an infant with congenital defects tell the nurse they will not come back to take their baby home. Which of the following actions should the nurse take next? 1. Determine why the parents will not pick their baby up. 2. Notify the physician so the physician can contact the parents. 3. Call the police to report an abandoned infant. 4. Refer the family to a social service agency.

1. The first action by the nurse would be to determine why the parents stated they would not pick up their baby.

A parent asks the nurse about head lice (pediculosis capitis) infestation during a visit to the clinic. Which of the following symptoms should the nurse tell the parent is most common in a child infected with head lice? 1) Itching of the scalp. 2) Scaling of the scalp. 3) Serous weeping on the scalp surface. 4) Pinpoint hemorrhagic spots on the scalp surface.

1. The most common characteristic of head lice infestation is severe itching.

The physician orders 250mg of an antibiotic every 6 hours for a child weighing 25kg who had infected burns. The normal dosage for this antibiotic and condition is 20 to 50 mg/kg per 24 hours. Which of the following actions would be most appropriate? 1. Carry out the order because the ordered dose is acceptable. 2. Give the dose recommended by the pharmacy reference material. 3. Question the order because the dose is too low. 4. Question the order because the dose is a toxic amount.

1. The ordered dose equals 1000 mg in 24 hrs. The recommended dose is 500 to 1250 mg in 24 hrs.

A nurse compares a child's height and weight with standard growth charts and finds the child to be in the 50th percentile for height and in the 45th percentile for weight. The nurse interprets these findings as indicating that the child is: 1) Average height and weight. 2) Overweight for height. 3) Underweight for height. 4) Abnormal in height.

1. The values of height and weight percentiles are usually similar for an individual child. Measurements between 5th and 95th percentiles are considered normal.

A mother brings her 2-year-old adopted Korean child to the clinic for an initial checkup. The child has been living with the adopted family for several weeks. The nurse notes an irregular area of deep blue pigment on the child's buttocks extending into the sacral area. The nurse should: 1. Do nothing concerning this finding. 2. Ask the mother in private how the bruise occurred. 3. Notify social services of a case of possible child abuse. 4. Question the mother about the family's discipline style.

1. This lesion is a mongolian spot, which is common in children of Asian or African American heritage.

A mother who brings her 4-month-old infant to the clinic for a regular checkup is concerned that her infant is not developing appropriately. When assessing the infant, which of the following should the nurse expect to find? 1) Sitting up with support. 2) Finger-to-thumb grasping. 3) Reaching for a toy. 4) Saying "mama" or "dada."

1. Typically a 4-month-old should be able to sit with support from a person holding the infant lightly in the area of the hips or lower chest.

A parent reports that his 2-year-old child often falls when running. The nurse interprets this as indicating which of the following as a normal aspect of a toddler's vision? 1. Nearsightedness. 2. Farsightedness. 3. Binocular vision. 4. Strabismus.

1. Until age 7 years, children are normally myopic (nearsighted).

When does a child throw a ball overhand?

18 months

The mother of a 6-month-old asks the nurse when her baby will get the first measles, mumps, and rubella (MMR) vaccine. Based on the recommended childhood immunization schedule published by the Centers for Disease Control, which response is accurate?

12 to 15 months

A nurse is doing discharge teaching. She explains that the anterior fontanel normally closes between ages:

12-18 months

Ampicillin, 75 mg/kg, is prescribed for a 22-lb child. It is available in a solution that contains 250 mg/5 mL. How many milliliters should the nurse administer in one dose?

15 Rationale: Take 22lbs / 2.2 = 10kg 10kg X 75mg/kg = 750mg 750/250 mg = 3mg X 5mL = 15

A nurse who is working in the Poison Control Center receives several telephone calls from parents whose children have ingested possible poisons. The nurse should recommend inducing vomiting for which child?

16-month old who drank 2 ounces of acetaminophen (Tylenol) elixir.

When does a child speak 2-3 word sentences?

2 Yeears

A parent whose family drinks low-fat milk asks if her child can begin to drink low fat milk. The AAP recommends that children can begin to drink low fat milk at what age?

2 years

When does a child achieve 50% of adult height?

2 years

The nurse is teaching an adolescent with asthma how to use an inhaler. In which order should the nurse instruct the client to follow the steps? 1) Inhale through an open mouth. 2) Breathe out through the mouth. 3) Hold the breath for 5 to 10 seconds. 4) Press the canister to release the medication.

2, 1, 4, 3.

The nurse is teaching the parents of a 5-year-old child who has just received diphtheria, tetanus, and pertussis; inactivated polio; and measles, mumps, and rubella vaccines about commonly expected adverse effects. What should be included? Select all that apply. 1. Fever of 103 F. 2. Redness at the injection site. 3. Rash. 4. Anorexia. 5. Prolonged crying. 6 Diarrhea.

2, 3, 4.

Toddlers

2-3 word sentences at 2 Ritualistic

A 16-year-old girl comes to the school nurse complaining of cramps, backache, and nausea with her periods. The nurse most likely would interpret these symptoms as which of the following? 1. Pathologic. 2. Physiologic. 3. Psychogenic. 4. Psychosomatic.

2.

A 6-month-old infant has a high fever and cold symptoms. She is pulling at her left ear. She is schedule to receive her 6-month immunizations. The mother asks the nurse if she will receive them. The nurse's best response would be: 1. She will receive just the hepatitis immunization today because she is so sick. 2. She can have them when she returns to have her ear rechecked. 3. She must be free of infection for 6 months before she can resume her immunizations. 4. She should have a pneumonia shot today instead.

2.

A mother expresses concern that picking up the infant whenever he cries will spoil him. What is the nurse's best response? 1. Allow him to cry for no longer than 45 minutes, then pick him up. 2. Babies need comforting and cuddling; meeting these needs will not spoil him. 3. Babies this young cry when they're hungry, try feeding him when he cries. 4. If it seems as if nothing is wrong, don't pick him up; the crying will stop eventually.

2.

Which of the following assessments would be the priority for a 2-year-old child after a bronchoscopy? 1. Cardiac rate. 2. Respiratory quality. 3. Sputum color. 4. Pulse pressure changes.

2.

An abused child is admitted to the hospital, and the nurse is aware that a court appearance may be necessary. To plan for this eventuality, what should be the priority? 1. Remembering the parent's and child's behavior when the child was admitted. 2. Documenting physical findings and behaviors observed during the child's admission. 3. Formulating subjective opinions about the cause of any injuries. 4. Preparing answers to questions that may be asked by the attorneys.

2.

At the day care center, one of the toddlers bites another child. Which of the following actions by the teacher would be most appropriate? 1. Bite the child who did the biting. 2. Place the child who did the biting in "time-out." 3. Spank the child who did the biting. 4. Call the parents to pick up the child who did the biting.

2.

The mother of a 4-year-old child is concerned about her child's masturbating. When responding to the mother, which of the following facts would the nurse need to keep in mind? 1. The child needs counseling for the abnormal behavior. 2. Masturbation is normal in children of this age. 3. The child is expressing some unmet needs. 4. Masturbation at this age provides sexual release.

2.

Which of the following instructions should the nurse include in the teaching plan about skin care for the mother of a child with atopic dermatitis? 1. Soaking the child in a tub for 30 minutes to soften the skin. 2. Using a mild soap followed by patting the skin to dry it. 3. Using an antibacterial soap two times a week. 4. Washing clothes in a strong detergent to prevent infections.

2.

Which of the following statements made by a mother of a 3-year-old child with unexplained injuries should the nurse determine as supportive of suspicions about abuse? 1. A good friend and I go shopping at least weekly. 2. I'm disappointed that my child can't tie his shoes. 3. My mother helps me with the children. 4. My child helps dress himself.

2.

Which of the following suggestions would be most appropriate in helping parents to prepare their children for starting school? 1. Have an older sibling tell the child about school. 2. Orient the child to the school's physical environment. 3. Offer to stay with the child for the first few days of school. 4. Discuss school with the child if he asks about it.

2.

After staying several hours with her 9-year-old daughter who is admitted to the hospital with an asthma attack, the mother leaves to attend to her other children. The child exhibits continued signs and symptoms of respiratory distress. Which of the following findings should lead the nurse to make a nursing diagnosis of Anxiety related to respiratory distress? 1. Complaints of an inability to get comfortable. 2. Frequent requests for someone to stay in the room. 3. Inability to remember her exact address. 4. Verbalization of a feeling of tightness in her chest.

2. A 9-year-old child should be able to tolerate being alone. Frequently asking for someone to be in the room indicates a degree of psychological distress that, at this age, suggests anxiety.

Which of the following actions would be most appropriate for a charge nurse to take first when finding that nurse who is caring for a very sick infant is making inappropriate remarks and acting in a bizarre manner? 1. Report this nurse to the supervisor. 2. Remove this nurse from the client assignment. 3. Call the nurse's family to have someone take the nurse home. 4. Talk with the nurse to determine why this behavior is occurring.

2. Because client safety is the priority, the most appropriate first action by the charge nurse would be to remove the nurse who is acting bizarrely from the client assignment.

A 5-month-old infant is brought to the clinic by his parents because he "cries too much" and "vomits a lot." The infant's birth weight was 6 lb, 10 oz, and his current weight is 7 lb, 4 oz, falling below the 5th percentile on a standard growth chart. Which of the following data should the nurse identify as the priority. 1. Frequency of regular check-ups/ 2. Feeding pattern. 3. Pattern of weight gain. 4. Family dynamics.

2. Because the infant falls below the 5th percentile on a standard growth chart, the nurse should consider failure to thrive.

The mother of a preschooler reports that her child creates a scene every night at bedtime. The nurse and the mother decide that the best course of action would be to do which of the following? 1) Allow the child to stay up later one or two nights a week. 2) Establish a set bedtime and follow a routine. 3) Encourage active play before bedtime. 4) Give the child a cookie if bedtime is pleasant.

2. Bedtime is often a problem with preschoolers. Recommendations for reducing conflicts at bedtime include establishing a set bedtime, having a dependable routine, such as story reading; and conveying the expectation that the child will comply.

After uncomplicated abdominal surgery, which of the following would be most appropriate when determining if an alert school-aged child is ready to drink oral fluids? 1. Ask if the child wants something to drink. 2. Auscultate the child's abdomen for bowel sounds. 3. Determine that the child has a gag reflex. 4. Palpate the epigastric area for discomfort.

2. Before giving fluids, the nurse needs to auscultate the child's abdomen for bowel sounds, which indicate the return of peristalsis and a functioning GI tract.

A child with appendicitis is being readied for surgery. What should be the nurse's first action? 1. Administer an enema. 2. Insert a nasogastric tube. 3. Obtain vital signs. 4. Administer antibiotics.

3.

Which of the following suggestions would be most helpful to the parents of a 2-year-old child when managing separation anxiety during hospitalization? 1. Leave while the child is sleeping. 2. Bring the child's favorite toys from home. 3. Tell the child the time they are leaving and returning. 4. Keep the visit time short.

2. Bringing a child's favorite toys, security blanket, or familiar objects from home can make the transition from home to hospital less stressful.

A 10-year-old child proudly tells the nurse that brushing and flossing her teeth is her responsibility. The nurse interprets this statement as indicating which of the following about the child? 1) She is too young to be given this responsibility. 2) She is most likely capable of this responsibility. 3) She should have assumed this responsibility much sooner. 4) She is probably just exaggerating the responsibility.

2. Children are capable of mastering the skills required for flossing when they reach 9 years of age. At this age, many children are able to assume responsibility for personal hygiene.

The mother of a child with tetralogy of Fallot asks the nurse why her child has clubbed fingers. The nurse bases the response on the understanding that clubbing is due to which of the following? 1. Anemia. 2. Peripheral hypoxia. 3. Delayed physical growth. 4. Destruction of bone marrow.

2. Clubbing of the fingers is one common finding in the child with persistent hypoxia, which leads to tissue changes in the body because of the low oxygen content of the blood.

Two adolescents come to the school nurse's office to talk about their friend. They are concerned because he seems to be using several different drugs. One of the adolescents asks how he would be able to tell if his friend was using cocaine. The nurse replies: 1. His eyes would be red and bloodshot. 2. His pupils would be large. 3. His pupils would be constricted to to pinpoints. 4. His eyes would look tired.

2. Cocaine use causes pupils to dilate.

A parent groupis discussing different types of punishment. The parents ask the nurse to discuss corporal punishment. The nurse tells the group that corporal punishment: 1. Does not physically harm the child. 2. Can result in children becoming accustomed to spanking. 3. Reinforces the idea that violence is not acceptable. 4. Can be beneficial in teaching children what they should do.

2. Corporal punishment is an aversion technique that teaches children what not to do.

The nurse explains to the mother of a child receiving digoxin (Lanoxin) that which of the following is the primary reason for giving this drug? 1. To relax the walls of the heart's arteries. 2. To improve the strength of the heartbeat. 3. To prevent irregularities in ventricular contractions. 4. To decrease inflammation of the heart wall.

2. Digitalis prepariations such as digoxin act to improve and strengthen the heartbeat.

Assessment of a 6-week-old infant reveals weight and length in the 50th percentile for his age and a head circumference at the 95th percentile. What should the nurse do first? 1. Assess motor and sensory function of the legs. 2. Examine the fontanels and sutures. 3. Advise the mother of the need for follow-up in 1 month. 4. Obtain a written consent for transillumination.

2. Head circumference usually parallels the percentile for length. The discrepancy found requires close and immediate attention because it could indicate hydrocephalus with its potential for brain damage.

The nurse notes that an infant stares at an object placed in her hand and takes it to her mouth, coos and gurgles when talked to, and sustains part of her own weight when held in a standing position. The nurse correctly interprets these findings as characteristics of an infant at which of the following stages? 1) 2months 2) 4 months 3) 7 months 4) 9 months

2. Holding the head erect while sitting, staring at an object placed in the hand,taking the object to the mouth, cooing and gurgling, and sustaining part of her body weight when in a standing position are behaviors characteristic of a 4-month-old infant.

Describe scissoring

a common characteristic of spastic cerebral palsy in infants; legs are extended and crossed over each other, feet are plantar flexed

Which of the following assessment findings should the nurse expect in an infant with colic? 1. Failure to gain weight. 2. Expulsion of flatus. 3. Soft abdomen. 4. Difficulty with burping.

2. Infants with colic have paroxysmal pain or cramping caused by the production and accumulation of gas.

Which of the following measures should the nurse expect to perform for a child who is receiving high-dose methotrexate (amethopterin) therapy? 1. Keeping the child in a fasting state. 2. Obtaining a while blood cell (WBC) count. 3. Preparing for radiography of the spinal canal. 4. Collecting a specimen for urinalysis.

2. Methotrexate is not highly toxic in low doses but may cause severe leukopenia at higher doses.

A father asks the nurse how he would know if his child had developed mononucleosis. The nurse explains that in addition to fatigue, which of the following would be most common? 1) Liver tenderness. 2) Enlarged lymph glands. 3) Persistent nonproductive cough. 4) A blush-like generalized skin rash.

2. Mononucleosis usually has an insidious onset with fatigue and the inability to maintain usual activity levels as the most common symptoms. The lymph nodes are typically enlarged and the spleen also may be enlarged.

Which of the following should the nurse do first when a neonate with myelomeningocele experiences urine retention with overflow incontinence? 1. Apply pressure to the suprapubic area. 2. Initiate an intermittent clean catheterization program. 3. Insert an indwelling urinary catheter. 4. Collect a urine specimen.

2. Overflow incontinence with constant dribbling is common in neonates with myelomeningocele.

A 2-year-old child brought to the clinic by her parents is uncooperative when the nurse tries to look in her ears. Which of the following should the nurse try first? 1) Ask another nurse to assist. 2) Allow a parent to assist. 3) Wait until the child calms down. 4) Restrain the child's arms.

2. Parents can be asked to assist when their child becomes uncooperative during a procedure. The child will feel more secure with a parent present.

Parents of a 15-year-old state that he is moody and rude. The nurse should advise his parents to: 1) Restrict his activities. 2) Discuss their feelings with their child. 3) Obtain family counseling. 4) Talk to other parents of adolescents.

2. Parents need to discuss with their adolescent how they perceive his behavior and how they feel about it.

The mother of a 4-year-old expresses concern that her child may be hyperactive. She describes the child as always in motion, constantly dropping and spilling things. Which of the following actions would be most appropriate at this time? 1) Determine whether there have been any changes at home. 2) Explain that this is not unusual behavior. 3) Explore the possibility that the child is being abused. 4) Suggest that the child be seen by a pediatric neurologist.

2. Preschool-age children have been described as powerhouses of gross motor activity who seem to have endless energy. A limitation of their motor ability is that in moving as quickly as they do, they are not always able to judge distances, nor are they able to estimate the amount of strength and balance needed for activities.

The school nurse develops a plan with an adolescent to provide relief of dysmenorrhea to aid in her development of which of the following? 1) Positive peer relations. 2) Positive self-identity. 3) A sense of autonomy. 4) A sense of independence.

2. Relieving dysmenorrhea in adolescence is crucial for the female's development of positive self-identity, of which positive body image and sexual identity are important components.

The mother tells the nurse that her 8--year-old child is continually telling jokes and riddles to the point of driving the other family members crazy. The nurse should explain this behavior is a sign of what? 1) Inadequate parental attention. 2) Mastery of language ambiguities. 3) Inappropriate peer influence. 4) Excessive television watching.

2. School-age children delight in riddles and jokes. Mastery of the ambiguities of language and of sentence structure allows the school-age child to manipulate words, and telling riddles and jokes is a way of practicing this skill.

Which of the following activities should the nurse include in the teaching plan for a mother to help channel her 4-year-old child's energy? 1. Participation in parallel play. 2. Play involving a game such as Simon Says. 3. Bicycle riding. 4. Stringing large beads.

2. Simon Says requires the preschooler to use a variety of motor skills, can help channel activity and meet developmental needs.

A mother asks the nurse, "How did my children get pinworms?" The nurse explains that pinworms are most commonly spread by which of the following when contaminated? 1) Food. 2) Hands. 3) Animals. 4) Toilet seats.

2. The adult pinworm emerges from the rectum and colon at night onto the perianal area to lay its eggs. Itching and scratching introduces the eggs to the hands, from where they can easily reinfect the child or infect others.

The father of a 3-year old child expresses a concern about the child's fear of the dark. Using Piaget's concepts of cognitive development which of the following would be the most appropriate explanation as the basis for the child's fear of darkness? 1. Reversibility. 2. Animism. 3. Conservation of matter. 4. Object permanence.

2. The child attributes the quality of conscious thought to inanimate objects. It is a peculiarity of preconceptual thought, part of the preoperational stage, lasting from age 2 to 4 years.

After teaching the mother of a toddler with iron deficiency anemia about diet modifications, the nurse determines that the teaching was initially effective when the mother verbalizes dietary changes involving which of the following? 1. Ingestion of equal amounts of iron-rich solids and milk products. 2. Increased intake of iron-rich solids and decreased milk intake. 3. Provision of several meals per day to the child. 4. Twice-daily offerings of dairy food snacks to the child.

2. The child's intake of iron-rich solids needs to be increased, while the intake of milk, which is low in iron, needs to be decreased to 1 quart per day.

The mother of a 6-month-old states that she has started her infant on 2% milk. which of the following should be the nurse's best response? 1) Your baby will probably be fine with this milk. 2) The baby should be switched to whole milk. 3) You need to keep the infant on formula. 4) You need to switch to formula right now.

2. The mother has already changed the infant from formula to cow's milk, so she probably will not change the infant back to formula. Therefore, the best the nurse can hope for is that the mother will switch to whole milk.

A child admitted to the hospital with a serum sodium level of 160 mmol/L is receiving 5% dextrose with 0.45 normal saline solution. The mother asks the child's nurse why the child is receiving sodium. The nurse's best reply would be: 1. Your child's sodium is high; I'll stop the infusion and check with the physician. 2. Your child's sodium is high; but if the serum sodium level is decreased too rapidly, it may cause seizures. 3. Your child's sodium is low; we need to give some more sodium I.V. 4. Your child's sodium is normal; the solution will maintain the level.

2. The normal serum sodium level for a child is 138-146 mmol/L. A rapid decrease in serum sodium level can cause fluid shifts that will result in a rapid increase in intracranial pressure, increasing the risk of seizure.

After having a blood sample drawn, a 5-year-old child insists that the site be covered with an adhesive bandage strip. When the mother tries to remove the bandage before leaving the office, the child screams that all the blood will come out. The nurse interprets this behavior as indicating a fear of which of the following? 1) Injury. 2) Compromised body integrity. 3) Pain. 4) Loss of control.

2. The preschool-age child does not have an accurate concept of skin integrity and can view medical and surgical treatments as hostile invasions that can destroy or damage the body. The child does not understand that exsanguination will not occur form an injection site.

An infant's skin is inelastic and the upper abdomen is distended. To palpate the olivelike mass most easily, the nurse should palpate the epigastrium just to the right of the umbilicus at which of the following times? 1. Just before the infant vomits. 2. While the infant is eating. 3. When the infant is lying on the left side. 4. When the stomach is empty

2. The pyloric, olivelike mass is most easily palpated when the abdominal muscles are relaxed, the stomach is empty and the infant is quiet. During eating, the stomach is still empty and the infant is relaxed and comfortable.

Which of the following actions initiated by the parents of an 8 month-old indicates they need further teaching about preventing childhood accidents? 1) Placing a fire screen in front of the fireplace. 2) Placing a car seat in a front-seat, front-facing position. 3) Inspecting toys for loose parts. 4) Placing toxic substances out of reach or in a locked cabinet.

2. The recommended safety-sear arrangement for infants up to 200 lb and less than 1 year old is rear-facing with shoulder restraints.

When planning a 15-month-old toddler's daily diet with the parents, which of the following amounts of milk should the nurse include? 1) 1/2 to 1 cup. 2) 2 to 3 cups. 3) 3 to 4 cups. 4) 4 to 5 cups.

2. Toddlers around the age of 15 months need 2 to 3 cups of milk per day to supply necessary nutrients such as calcium.

When assessing for pain in a toddler, which of the following methods should be the most appropriate? 1) Ask the child about the pain. 2) Observe the child for restlessness. 3) Use a numeric pain scale. 4) Assess for changes in vital signs.

2. Toddlers usually express pain through such behaviors as restlessness, facial grimaces, irritability, and crying.

An 8 month-old infant is seen in the well-child clinic for a routine checkup. The nurse should expect the infant to be able to do which of the following. Select all that apply. 1) Say "mama" and "dada" with specific meaning. 2) Feed self with spoon. 3) Play peek-a-boo. 4) Walk independently. 5) Stack two blocks. 6) Transfer object from hand to hand.

3, 6. Typically abilities demonstrated by 8-month-old infants include peek-a-boo and transferring objects from one hand to another.

A mother, concerned about her infant's surgery for inguinal hernia repair, asks the nurse if her infant would have been scheduled for surgery even if the hernia had been asymptomatic. Which of the following statements offers the best explanation of why the surgical repair should be done at this time? 1. An infant is better able to tolerate the physical stress of surgery than an older child is. 2. The experience of surgery is less frightening for the younger child. 3. Less danger and fewer complications result when surgery is an elective procedure. 4. Doing surgery near the genital organs is preferred before a child becomes conscious of sexual identity.

3.

A nurse overhears a fellow staff member talking about the mother of a child for whom the staff nurse is caring. The nurse is telling others private information that the mother had shared. Which of the following responses by the nurse overhearing the conversation would be best? 1. Reporting this incident to their nurse-manager. 2. Telling the mother what was being said about her. 3. Talking to the staff member privately about this. 4. Talking to the staff in general about confidentiality.

3.

As part of the annual health screening, the nurse visits the eight-grade physical education classes and asks each student to bend forward at the waist with the back parallel to the floor and the hangs together at midline. For which of the following is the nurse assessing? 1. Slipped epiphysis. 2. Developmental dysplasia of hip. 3. Idiopathic scoliosis. 4. Physical dexterity.

3.

For the child experiencing excessive vomiting secondary to pyloric stenosis, the nurse should assess the child for which of the following acie-base imbalances? 1. Respiratory alkalosis. 2. Respiratory acidosis. 3. Metabolic alkalosis. 4. Metabolic acidosis.

3.

The parents of a neonate with a cleft lip are shocked when they see their child for the first time. Which of the following nursing actions should the nurse include in the neonate's plan of care to help the parents accept their infant's anomaly? 1. Encouraging the parents to visit more frequently. 2. Reassuring them that surgery will correct the defect. 3. Showing them pictures of babies before and after corrective surgery. 4. Allowing them to complete their grieving process before seeing the infant again.

3.

What is appropriate to include in a teaching plan for a 9-year-old child who has had diabetes for several years? 1. Beginning to recognize the signs and symptoms of hypoglycemia. 2. Learning to measure insulin accurately in a syringe. 3. Beginning to be able to self-administer injections with adult supervision. 4. Assuming responsibility for self care.

3.

Which of the following discharge instructions should the nurse give the parents of an infant with a temporary colostomy? 1. Flush the stoma with tap water at least once a day. 2. Allow the diaper to absorb the colostomy drainage. 3. Give the infant plenty of liquids to drink. 4. Expect the stoma to become dusky red within 2 weeks.

3.

Which of the following would be best to help prepare a preschool-aged child for an injection? 1. Having an older child explain that shots do not hurt. 2. Helping the child to imagine she is in a different place. 3. Giving the child a play syringe and a bandage to give a doll injections. 4. Giving the child a pounding board to encourage expressions of anger.

3.

When discussing the onset of adolescence with parents, the nurse explains that it occurs at which of the following times? 1) Same age for both boys and girls. 2) 1 to 2 years earlier in boys than in girls. 3) 1 to 2 years earlier in girls than in boys. 4) 3 to 4 years later in boys than in girls.

3. Girls experience the onset of adolescence about 1 to 2 years earlier than boys.

When assessing a 2-year-old child brought by his mother to the clinic for a routine checkup, which of the following should the nurse expect the child to be able to do? 1) Ride a tricycle. 2) Tie his shoelaces. 3) Kick a ball forward. 4) Use blunt scissors.

3. A 2-year old child usually can kick a ball forward.

A mother of a toilet-trained 3-year-old expresses concern over her child's bedwetting while hospitalized. The most appropriate response for the nurse to make is to tell the mother: 1) He was too immature to be toilet trained. In a few months he should be old enough. 2) Children are afraid in the hospital and frequently wet their bed. 3) It's very common for children to regress when they're in the hospital. 4) This is normal. He probably received too much fluid the night before.

3. A child will regress to a behavior used in an earlier stage of development in order to cope with a perceived threatening situation.

A mother calls the clinic to talk to the nurse. The mother states that a physician described her daughter as having 20/60 vision and she asks the nurse what this means. The nurse responds based on the interpretation that the child is experiencing which of the following? 1) A loss of approximately one-third of her visual acuity. 2) Ability to see at 60 feet what she should see at 20 feet. 3) Ability to see at 20 feet what she should see at 60 feet. 4) Visual acuity three times better than average.

3. A child with 20/60 vision sees at 20 feet what those with 20/20 vision see at 60 feet.

A mother brings her 18-month-old to the clinic because the child "eats ashes, crayons, and paper." Which of the following information about the toddler should nurse assess first? 1) Evidence of eruption of large teeth. 2) Amount of attention from the mother. 3) Any changes in the home environment. 4) Intake of a soft, low-roughage diet.

3. A craving to eat nonfood substances is known as pica. Toddlers use oral gratification as a means to cope with anxiety. Therefore, the nurse should first asses whether the child is experiencing any change in the home environment that could cause anxiety.

A mother brigs her child to the emergency department after the child has taken "some white pills just a short while ago." What should lead the nurse to determine that the pills taken were most probably acetaminophen? 1. Nosebleed. 2. Seizure activity. 3. Nausea and vomiting. 4. Deep, rapid respirations.

3. Acetaminophen is a common drug poisoning agent in children. Symptoms seen in the first 4 hours include nausea and vomiting, anorexia, malaise, and palor.

A nurse is assessing the growth and development of a 14-year-old boy. He reports that his 13-year-old sister is 2 inches taller than he is. The nurse should advise the boy that the growth spurt in adolescent boys, compared with the growth spurt of adolescent girls: 1) Occurs at the same time. 2) Occurs 2 years earlier. 3) Occurs 2 years later. 4) Occurs 1 year earlier.

3. Adolescent boys lag about 2 years behind adolescent girls in growth.

The nurse discusses the eating habits of school-age children with their parents, explaining that these habits are most influenced by which of the following? 1) Food preferences of their peers. 2) Smell and appearance of foods offered. 3) Examples provided by parents at mealtimes. 4) Parental encouragement to eat nutritious foods.

3. Although children may be influenced by their peers and smell and appearance of foods may be important, children are most likely to be influenced by the example and atmosphere provided by their parents.

A 13-month-old child has just been placed in a plaster hip spica cast to correct a congenital anomaly. Which nursing actions should be included in the plan of care? 1. Turn the child no more than every four hours to minimize manipulation of the wet cast. 2. Use only fingertips when moving the child to prevent indentations in the cast. 3. Assess and document neurovascular function at least every two hours. 4. Use a hair dryer to speed the cast-drying process.

3. Assess and document neurovascular function

To encourage autonomy in a 4 year-old, the nurse should instruct the mother to: 1) Discourage the child's choice of clothing. 2) Button the child's coat and blouse. 3) Praise the child's attempts to dress herself. 4) Tell the child when the combination of clothes is not appropriate.

3. At age 4, the child should be learning to dress without supervision. A child will feel more autonomous if allowed to try to take on tasks herself. Such attempts should be encouraged to increase self-esteem. Allowing choices encourages the child's capacity to control her behavior.

A 2-year-old tells his mother he is afraid to go to sleep because "the monsters will get him." The nurse should tell his mother to: 1) Allow him to sleep with his parents in their bed whenever he is afraid. 2) Increase his activity before he goes to bed, so he eventually falls asleep from being tired. 3) Read a story to him before bedtime and allow him to have a cuddly animal or a blanket. 4) Allow him to stay up an hour later with the family until he falls asleep.

3. Behavior problems related to sleep and rest are common in young children. Consistent rituals around bedtime help to create an easier transition from waking to sleep.

The nurse is caring for a child who has Duchenne's muscular dystrophy. What understanding is correct about the progress of the disease? 1. The disease is controllable with aggressive treatment. 2. Most children will die of something else before they die of muscular dystrophy. 3. Brothers of children with muscular dystrophy should be evaluated for the disease. 4. Muscular dystrophy causes its victims to become incoherent and often violent.

3. Brothers of children with muscular dystrophy

After teaching a group of parents of preschoolers attending a well-child clinic about oral hygiene and tooth brushing, the nurse determines that the teaching has been successful when the parents state that children can begin to brush their teeth without help at which of the following ages? 1) 3 years. 2) 5 years. 3) 7 years. 4) 9 years.

3. Children younger than 7 years of age do not have the manual dexterity needed for tooth brushing. Therefore, parents need to help with this task until that time.

After the nurse instructs the parents of a 5-month-old infant about the purpose of the Denver Developmental Screening Test (DDST), which of the following statements by the parents about what the test measures would indicate that the teaching was effective? 1. This test measures a child's IQ. 2. This test measures a child's emotional development. 3. This test measures a child's social and physical abilities. 4. This test measures a child's potential for future development.

3. DDST measures a child's social, language, and fine and gross motor skills by testing abilities that usually occur at a given age.

What intervention should the nurse perform first in caring for a child who has ingested a poison?

assessment of the child's respiratory, cardiac, and neurological status

A mother is concerned because her 5-year-old son seems prone to minor accidents such as skinning his elbows and knees and falling off his scooter. The nurse explains to the mother that childhood accidents are more likely to occur in which of the following situations? 1) The child is the sole child in the family. 2) The family has limited formal education. 3) The family is experiencing changes. 4) The child and family live in the suburbs.

3. Family changes and stresses (e.g. moving, having company, taking vacations, adding new members) can distract parental attention and contribute to accidents.

Which of the following methods should the nurse use to feed an infant after surgical repair of a cleft lip? 1. Gastric gavage. 2. I.V. fluids. 3. Bottle with a cross-cut nipple. 4. Bottle with a lamb's nipple.

3. Feeding methods should produce the least ension possible on the sutures to promote effective healing of the cleft lip repair.

After the nurse assesses a 2 1/2 year-old's teeth during the physical examination, which of the following instructions should the nurse give to the mother? 1. Make sure the child brushes his teeth after every meal and at bedtime. 2. Give the child a small, soft-bristled toothbrush to use. 3. Floss the child's teeth using dental floss. 4. Add a fluoride supplement to the child's milk.

3. For a toddler, a parent should clean and floss the toddler's teeth because the child does not have the cognitive or motor skills needed for effective cleaning.

A child with Down syndrome has an IQ of about 40. The nurse should expect which of the following as the type of environment and interdisciplinary program to most likely benefit this child? 1. Custodial. 2. Institutional. 3. Task analysis. 4. Vocational training.

3. Habit-training task analysis, a step-by-step process in which each step is taught before moving onto the next step, would be most beneficial.

Several high-school seniors are referred to the school nurse because of suspected alcohol misuse. When the nurse assesses the situation, what would be most important to determine? 1) What they know about the legal implications of drinking. 2) The type of alcohol they usually drink. 3) The reasons they choose to use alcohol. 4) When and with whom they use alcohol.

3. Information about why adolescents choose to use alcohol or other drugs can be used to determine whether they are becoming responsible users or problem users.

Which of the following is appropriate language development for an 8-month old? The child should be: 1) saying "dada" and "mama" specifically ("dada" to father and "mama" to mother). 2) saying three other words besides "mama" and "dada." 3) saying "dada" and "mama" nonspecifically. 4) saying "ball" when parents point to a ball.

3. It is important for the nurse to assist parents in assessing speech development in their child so that developmental delays can be identified early. At 8 months of age, the child should say "mama" and "dada" nonspecifically and imitate speech sounds.

A 10-year-old child with a history of bronchial asthma triggered by exposure to cold, smoke, and nuts is brought to the hospital's emergency department by his mother. Appearing restless and anxious, the child has a respiratory rate of 36 breaths/minute and pulse rate of 160 beats/minute. Which of the following findings should be of greatest concern to the nurse? 1. Increased respiratory effort. 2. Moist, loose cough. 3. Absence of wheezing. 4. Prolonged expiratory phase.

3. Knowing that this child is most likely experiencing an asthma attack, the nurse should expect to hear wheezing and note some shortness of breath with a prolonged expiratory phase.

A parents asks, "Can I get head lice too?" The nurse indicates that adults can also be infested with head lice but that pediculosis is more common among school children, primarily for which of the following reasons? 1) An immunity to pediculosis usually is established by adulthood. 2) School-age children tend to be more neglectful of frequent handwashing. 3) Pediculosis usually is spread by close contact with infested children. 4) The skin of adults is more capable of resisting the invasion of lice.

3. Lice are spread by close personal contact and by contact with infested clothing, bed and bathroom linens, and combs and brushes.

Initiation of which of the following immunizations is recommended prior to the adolescent entering college? 1) Diphtheria, tetanus, and acellular pertussis (DTaP). 2) Varicella. 3) Meningococcal. 4) Pneumococcal conjugate vaccine (PCV).

3. Meningococcal vaccine should be administered before the adolescent enters college because outbreaks of this type of meningitis are likely when people live in close association, such as in college dorms.

After teaching the parents of an infant who has had a pyloromyotomy about proper postoperative feeding techniques, the nurse determines that they have understood the teaching when they position the infant in the crib after feeding with head elevated and lying on: 1. Left side. 2. Abdomen. 3. Right side. 4. Back.

3. Positioning the infant on the right side with the head elevated facilitates passage of food through the pyloric sphincter in to the intestine.

When developing the teaching plan about illness for the mother of a preschooler, which of the following should the nurse include about how a preschooler perceives illness? 1) A necessary part of life. 2) A test of self-worth. 3) A punishment for wrong-doing. 4) The will of God.

3. Preschool-age children may view illness as punishment for their fantasies. At this age children do not have the cognitive ability to separate fantasies from reality and may expect to be punished for their "evil thoughts."

A mother asks the nurse when she should wean her 4-month-old infant from breast feeding and begin using a cup. What should the nurse explain as the best indication of the infant's readiness to be weaned? 1. Taking solid foods well. 2. Sleeping through the night. 3. Shortening the nursing time. 4. Eating on a regular schedule.

3. Readiness for weaning is an individual matter but is usually indicated when an infant begins to decrease the time spent nursing.

When obtaining a health history from the mother of a 7-year-old child diagnosed with acute rheumatic fever, the nurse should focus questions to determine if the child was recently ill with which of the following? 1. Vomiting. 2. Earache. 3. Sore throat. 4. Dysuria.

3. Rheumatic fever is an inflammatory collagen disease that typically follows an infection by group A beta-hemolytic streptococci, ordinarily occurring in the throat.

After teaching the parents about the cause of ringworm of the scalp (tinea capitis), which of the following, if stated by the father, indicates successful teaching? 1) It results from overexposure to the sun. 2) It's caused by infestation with a mite. 3) It's a fungal infection of the scalp. 4) It's an allergic reaction.

3. Ringworm of the scalp is caused by a fungus of dermatophyte group of the species.

The mother asks the nurse about her 9-year-old child's apparent need for between-meal snacks, especially after school. When developing a sound nutritional plan for the child with the mother, which of the following should the nurse need to keep in mind? 1) The child does not need to eat between-meal snacks. 2) The child should eat the snacks the mother thinks are appropriate. 3) The child should help with preparing his or her own snacks. 4) The child will instinctively select nutritional snacks.

3. Snacks are necessary for school-age children because of their high energy level. School-age children are in a stage of cognitive development in which they can learn to categorize or classify and can also learn cause and effect.

A nurse observes a family in the waiting room of a well-child clinic. Which of the following behaviors would be considered to be an example of social affective play? 1. An 8-year-old child is taking turns playing a handheld video game with another child. 2. A 4-year-old child is listening to the mother's chest with a stethoscope. 3. An infant is making happy noises in response to her father speaking to her. 4. A 2-year-old child is sitting in her mother's lap hugging a teddy bear.

3. Social affective play occurs when infants take pleasure in relationships with people.

The mother of a 2-year-old is concerned because the child's right eye seems to turn in toward his nose when he is tired. The nurse should: 1) Assure the mother that this is a normal event when the child is tired. 2) Advise the mother to continue to watch his eyes closely and if the problem persists to call the clinic. 3) Test the child with the cover-uncover test and refer the mother and child to an ophthalmologist if the test is abnormal. 4) Explain to the mother that the child will probably outgrow the weakness and she need not be concerned.

3. Strabismus is diagnosed through observation and use of the corneal light reflex test. The cover-uncover test will reveal movement of the affected eye when the unaffected eye is covered, indicating abnormal fixation of the affected eye.

A mother tells the nurse that one of her children has chickenpox and asks what she should do to care for that child. When teaching the mother, which of the following would be most important to prevent? 1) Acid-base imbalance. 2) Malnutrition. 3) Skin infection. 4) Respiratory infection.

3. The care of a child with chickenpox focuses primarily on preventing infection in the lesions. The lesions cause severe itching, and organisms are ordinarily introduced into the lesion through scratching.

A 14-year-old boy brought to the emergency department complaining of right lower quadrant pain is tentatively diagnosed with acute appendicitis. When assessing the boy, what should the nurse expect to find? 1. Costovertebral angle tenderness. 2. Widening pulse pressure. 3. Oral temperature of 100 F. 4. Gross hematuria.

3. The most common manifestations of appendicitis include right lower quadrant pain, localized tenderness, and a fever of 99 F to 102 F.

Which of the following behaviors by a neonate attempting an initial feeding should indicate to the nurse that the neonate may have tracheoesophageal fisula? 1. Sucking attempts that are too poorly coordinated to effective. 2. Projectile vomiting that occurs after drinking 4 oz. 3. Coughing, choking, and cyanosis that occur after several swallows of formula. 4. Sleeping that occurs after taking 10ml of formula with an inability to be stimulated to take more.

3. The newborn with tracheoesophageal fistula swallows normally, but the fluids quickly fill the blind pouch.

During a home visit, the nurse notices that a 1-month-old infant has esotropia. The nurse should advise the parents to do which of the following? 1. Call the baby's health care provider immediately. 2. Mention this finding at the baby's 6-month checkup. 3. Do nothing because this condition is normal for the infant's age. 4. Call the clinic for a referral to an optometrist.

3. The nurse should advise the parents to do nothing because esotropia, inward turning of the eyes, is a normal finding in infants of this age.

An adolescent tells the school nurse that she would like to use tampons during her period. Which of the following would be most appropriate for the nurse to do? 1) Assess her usual menstrual flow pattern. 2) Determine whether she is sexually active. 3) Provide information about preventing toxic shock syndrome. 4) Refer her to a specialist in adolescent gynecology.

3. The nurse should provide the adolescent with information about toxic shock syndrome because of the identified relationship between tampon use and the syndrome's development.

Which of the following structures should be closed by the time the child is 2 months old? 1) Anterior fontanel 2) sagittal suture 3) Posterior fontanel 4) Frontal suture

3. The posterior fontanel should be closed by age 2 months. The anterior fontanel and sagittal and frontal sutures should be closed by age 18 months.

After having surgery to reduce the invagination of intussusception, an infant has a nasogastric tube in place, is receiving I.V. fluids, and is allowed nothing by mouth. In addition to body weight, which of the following parameters should the nurse use to calculate the amount of I.V. fluid and electrolyte solution to infuse over the next 24 hours? 1. Stool output. 2. Urine output. 3. Gastric output. 4. Degree of temperature elevation.

3. The volume of parenteral fluids needed is based on fluid requirements determined according to body weight and, in this situation, gastric output. If these fluids are not replaced with an appropriate I.V. solution, serious fluid and electrolyte imbalances could develop.

A mother tells the nurse that her 4 1/2-year-old child "doesn't seem to know the difference between right and wrong." The nurse responds to the mother, basing the explanation on the fact that this behavior is typical of which of the following levels as described by Kohlberg's theory of levels of moral development? 1. Autonomous. 2. Conventional. 3. Preconventional. 4. Principled.

3. This stage is typical of the preschool-aged child.

When preparing to admit an infant diagnosed with diarrhea to the pediatric unit, the nurse should expect to assign the infant to which of the following rooms? 1. A four-bed room with postoperative clients. 2. A two-bed room with an infant with respiratory disease. 3. A two-bed room with no roommate. 4. A room with other infants younger than age 1 year.

3. To reduce the risk of infection transmission, an infant with diarrhea of undetermined origin should be placed in a room alone until a causative organism can be identified.

a 10-month old looks for objects that have been removed from his view. The nurse should instruct the parents that: 1) Neuromuscular development enables the child to reach out and grasp objects. 2) The child's curiosity has increased. 3) The child understands the permanence of objects even though the child cannot see them. 4) The child is now able to transfer objects from hand to hand.

3. Understanding object permanence means that the child is aware of the existence of objects that are covered or displaced.

An adolescent tells the nurse that he would like to lose weight and asks the nurse's opinion on how to accomplish his goal. Which of the following suggestions would be most appropriate? 1. Exercising more often. 2. Severely limiting calorie intake. 3. Participating in an adolescent weight-reduction program. 4. Cutting down on sweets and other snacks.

3. Weight loss treatment modalities that include peer involvement have been proven to be the most successful approach with obese adolescents.

A mother of an ill child is concerned because the child "isn't eating well." Which of the following strategies devised by the mother to help increase the child's intake should the nurse advise against using? 1. Allowing the child to choose his meals from an acceptable list of foods. 2. Letting the child substitute items on his tray for other nutritious foods. 3. Asking the child to say why he is not eating. 4. Telling the child he must eat or else he will not get better.

4.

A nurse caring for a 15-month old girl suspects that she has been sexually abused. What rule should guide the nurse to the decision to report the abuse? 1. The parents need to be notified before suspected abuse can be reported. 2. Physicians are primarily responsible for reporting suspected abuse. 3. A nurse can be sued when reporting abuse on suspicions only. 4. A nurse who suspects child abuse is legally required to report the suspicions.

4.

A parent says that her family will soon be traveling abroad and asks why the drinking water in many regions must be boiled. The nurse should explain that, in addition to various types of dysentery, contaminated drinking water is most commonly responsible for the transmission of which disease? 1. Yellow fever. 2. Brucellosis. 3. Poliomyelitis. 4. Typhoid fever.

4.

The mother of a 4-year-old child asks about dental care for her child. "I help brush her teeth every day, and her teeth look healthy," the mother states. "When should I take her to see a dentist?" Which of the following responses would be most appropriate? 1. Because you help brush her teeth, there's no need to see a dentist right now. 2. Ideally she should have seen a dentist already, but it's still not too late. 3. Your child doesn't need to see the dentist until she starts school. 4. A dental checkup is a good idea even if no problems are noticeable.

4.

The mother says that the infant's physician recommends certain foods, but the infant refuses to eat them after breast-feeding. The nurse should suggest that the mother alter the feeding plan by doing which of the following? 1. Offering dessert followed by some vegetables and meat. 2. Offering breast milk as long as the infant refuses to eat solid foods. 3. Mixing pureed food with some breast milk in a bottle with a large-hole nipple. 4. Allowing the infant to nurse for a few minutes and then offering solid foods.

4.

When caring for terminally ill children and their families, which of the following is recommended as most important for the nurse to have? 1. Experience with the death of a loved one. 2. Development of a belief that accepts life after death. 3. Participation in a course examining how best to deal with death and grieving. 4. A working personal philosophy concerning life and death.

4.

When completing an assessment of a healthy adolescent client, which of the following would be most appropriate? 1. Obtain a detailed account of the adolescent's prenatal and early developmental history. 2. Discuss sexual preferences and behaviors with the parents present for legal reasons. 3. Discuss the client's smoking with parents present in the room. 4. Gather information from the parents and adolescent; then assess the adolescent in private.

4.

While planning interventions with the nurse that will allow the diabetic child to participate in an early morning tennis program at school, the mother offers several interventions. What should the nurse recommend eliminating? 1. Injecting the morning insulin dose in an area away from major muscles used in playing tennis. 2. Having the child eat more calories for breakfast on tennis days. 3. Having the child carry a source of quickly absorbed carbohydrate to the program. 4. Teaching the other children in the class the signs and symptoms of hyperglycemia.

4.

The mother of a 1-month-old infant states that she is curious as to whether her infant is developing normally. Which of the following developmental milestones should the nurse expect the infant to perform? 1) Smiling and laughing out loud. 2) Rolling from front to side. 3) Holding a rattle briefly. 4) Turning the head from side to side.

4. A 1-month-old infant usually is able to lift the head and turn it from side to side when lying prone.

The nurse should refer the parents of an 8-month-old child to a health care provider if the child is unable to: 1) stand momentarily without holding onto furniture. 2) Stand alone well for long periods of time 3) Stoop to recover an object. 4) Sit without support for long periods of time.

4. A child of 8 months should sit without support for ling periods of time. His muscles are not developed enough to support all his weight without assistance.

In addition to immunizing for diphtheria, tetanus, and acellular pertussis (DTaP) during the first 6 months of life, the nurse should administer which of the following immunizations? 1) Mumps. 2) Measles. 3) Tuberculosis. 4) Hepatitis B.

4. A series of three injections of DTaP and a series of three injections of Hib vaccine are recommended during the first year of life. In addition the infant should receive three immunizations for hepatitis B.

After an appendectomy, an adolescent is alert and oriented. Parenteral fluids are infusing and a nasogastric tube is attached to low intermittent suction. Which of the following nursing measures would be most appropriate for the adolescent during this early postoperative period? 1. Irrigating the nasogastric tube every hour. 2. Testing the urine for protein. 3. Removing the nasogastric tube when the adolescent is fully alert. 4. Encouraging the adolescent to urinate frequently.

4. After an appendectomy, the adolescent should be encouraged to void frequently to prevent bladder distention which could cause strain on the incision.

After surgical repair of a cleft lip, an infant exhibits difficulty breathing. Which of the following measures should the nurse institute first? 1. Raising the infant's head. 2. Turning the infant onto the abdomen. 3. Administering oxygen by mask. 4. Exerting downward pressure on the infant's chin.

4. After the repair of a cleft lip, the infant must become accustomed to nasal breathing. If the infant is having difficulty breathing, it would be best to open the mouth by exerting downward pressure on the chin.

The school nurse is invited to attend a meeting with several parents who express frustration with the amount of time their adolescents spend in front of the mirror and the length of time it takes them to get dressed. The nurse explains that this behavior indicates: 1) An abnormal narcissism. 2) A method of procrastination. 3) A way of testing the parents' limit-setting. 4) A result of developing self-concept.

4. An adolescent's body is undergoing rapid changes. Adolescence is a time of integrating these rapidly occuring physical changes into the self-concept to achieve the developmental task of a positive self-identity.

When talking with grandparents of a toddler, which of the following toys should the nurse recommend as the most appropriate? 1. Tricycle. 2. Wheelbarrow. 3. Sled. 4. Blocks.

4. As toddlers begin imaginative play, blocks are an excellent toy choice.

A young child who has undergone a tonsillectomy refuses to let the nurse look at the tonsillar beds to check for bleeding. To assess whether the child is bleeding from the tonsillar beds, which of the following would be most appropriate? 1. Assess capillary refill. 2. Force open the mouth with a tongue blade. 3. Monitor for decreased blood pressure. 4. Observe for frequent swallowing.

4. Blood will go down the back of the throat causing the child to swallow frequently.

The nurse asks a 9-year-old child and her mother about the child's best friend to assess which of the following about the child? 1) Language development. 2) Motor development. 3) Neurologic development. 4) Social development.

4. During the school-age years, a child learns to socialize with children of the same age. Therefore, the nurse is assessing the child's social development.

After teaching the parents of a 15-month-old child who has undergone cleft palate repair how to use elbow restraints, which of the following statements by the parents indicates effective teaching? 1. We'll keep the restraints in place continuously until the doctor says it's okay to remove them. 2. We can take off the restraints while our child is playing but we'll make sure to put them back on at night. 3. The restraints should be taped directly to our child's arms so that they will stay in one place. 4. We'll remove the restraints temporarily at least three times a day to check his skin then put them right back on.

4. Elbow restraints help to keep the child from placing fingers or any other object in the mouth that would cause injury to the operative site.

Griseofulvin (Grisactin) was ordered to treat a child's ringworm of the scalp. The nurse instructs the parents to use the medication for several weeks for which of the following reasons? 1) A sensitivity to the drug is less likely if it is used over a period of time. 2) Fewer side effects occur as the body slowly adjusts to a new substance over time. 3) Fewer allergic reactions occur if the drug is maintained at the same level long-term. 4) The growth of the causative organism into new cells is prevented with long-term use.

4. Griseofulvin is an antifungal agent that acts by binding to the keratin that is deposited in the skin, hair and nails as they grow. This keratin is then resistant to the fungus. But as the keratin is normally shed, the fungus enters new, uninfected cells unless drug therapy continues.

A nurse working in the nursery identifies a goal for a mother of a newborn to demonstrate positive attachment behaviors upon discharge. Which intervention would be least effective in accomplishing this goal? 1) Provide opportunities for the mother to hold and examine the newborn. 2) Engage the mother in the newborn's care. 3) Create an environment that fosters privacy for the mother and newborn. 4) Identify strategies to prevent difficulties in parenting.

4. Identifying ways to prevent difficulties in parenting would be helpful in reducing the incidence of child abuse and reducing the stress of child rearing.

When observing the parent instilling prescribed ear drops ordered twice a day for a toddler, the nurse decides that the teaching about positioning of the pinna for instillation of the drops is effective when the parent pulls the toddler's pinna in which of the following directions? 1) Up and forward. 2) Up and backward. 3) Down and forward. 4) Down and backward.

4. In a child younger than 3 years of age, the pinna is pulled back and down, because the auditory canals are almost straight in children.

The nurse is teaching the parents of an 8-month-old about what the child should eat. The nurse should include which of the following points in the teaching plan? 1) Items from all four food groups should be introduced to the infant by the time the child is 10 months old. 2) Solid foods should not be introduced until the infant is 10 months old. 3) Iron deficiency rarely develops before 12 months of age, so iron-fortified cereals should not be introduced until the infant is 12 months old. 4) The infant's diet can be changed from formula to whole milk when the infant is 12 months old.

4. Infants should be kept on formula or breast milk until 1 year of age. the protein in cow's milk is harder to digest than that found in formula.

The parents of teenagers express concerns about the types and large quantities of food their children eat and their refusal to eat foods served at family meals. Which of the following suggestions would be most helpful for the parents? 1. Carefully evaluate the adolescents' nutritional intake. 2. Inform the adolescents about the adverse effects of fad diets. 3. Give the adolescents responsibility for grocery shopping for 1 month. 4. Incorporate the adolescents' preferences into meal planning.

4. Preventing food intake from becoming the center of an independence-dependence struggle is important.

A nurse is assessing the growth and development of a 10-year old. What is the expected behavior of this child? 1) Enjoys physical demonstrations of affection. 2) Is selfish and insensitive to the welfare of others. 3) Is uncooperative in play and school. 4) Has a strong sense of justice and fair play.

4. School-age children are concerned about justice and fair play.

Which of the following demonstrates the nurse's compliance with the Centers for Disease Control and prevention guidelines concerning sterile glove use? 1. As an optional precautionary measure. 2. When delivering care involving touching a child. 3. Upon entering a child's room. 4. When giving direct care to burned areas.

4. Sterile gloves must be worn when giving any care to a burn area.

A parent seems concerned about the fact that the infant's soft spot is still open. Which of the following should the nurse include when explaining about the usual age for closure of the soft spot near the front of the infant's head. 1) 2-4 months 2) 5-8 months 3) 9-11 months 4) 12-18 months

4. The anterior fontanel, the soft spot near the front of the infant's head, usually closes between 12-18 months.

How is a child usually positioned after brain tumor surgery?

Flat or on either side

During a routine health assessment, a mother tells the nurse that her 2-year old child is using a potty seat but is still having problems toilet training. Which of the following suggestions would be most appropriate? 1. Offer the child more praise each time. 2. Use a potty chair instead of a potty seat. 3. Focus on the "accidents" that occur during training. 4. Defer training until the child is developmentally ready.

4. The most common reason for failed toilet training is that the child is simply not developmentally ready for training.

The nurse is discharging from the hospital an 8-month-old who weighs 15 lb. The parents have put the child in the back seat of the car with the car seat facing the front seat. The nurse should: 1) Ask the parents to wait while the nurse obtains the correct car seat. 2) Complete the discharge with the child facing the front seat. 3) Give the parents a manual on proper car seat placement. 4) Show the parents proper placement of the car seat facing the back seat.

4. The proper placement for a car seat for a child less than 20 lb and younger than 1 year is in the back seat, facing the rear of the car.

When administering an I.M. injection to a neonate, which of the following muscles should the nurse consider as the best injection site? 1. Deltoid. 2. Dorsogluteal. 3. Ventrogluteal. 4. Vastus Lateralis.

4. There is less danger of injuring nerves, blood vessels, or bony structures at this site.

The mother asks the nurse for advice about discipline for her 18-month-old. Which of the following should the nurse suggest that the mother use first? 1) Structured interactions. 2) Spanking. 3) Reasoning. 4) Time-out.

4. Time out is the most appropriate discipline for toddlers. It helps to remove them from the situation and allows them to regain control.

A 2-year-old always puts his teddy bear at the head of his bed before he goes to sleep. The parents ask the nurse if this behavior is normal. The nurse should explain to the parents that toddlers use ritualistic patterns to: 1) Establish a sense of identity. 2) Establish control over adults in their environment. 3) Establish sequenced patterns of learning behavior. 4)Establish a sense of security.

4. Toddlers establish ritualistic patterns to feel secure, despite inconsistencies in their environment.

A 5-year-old child asks the nurse if it will hurt to have his tonsils and adenoids taken out. Which of the following responses by the nurse would be best? 1. It won't hurt because we put you to sleep. 2. It won't hurt because you're such a big boy. 3. It will hurt because of the incisions made in the throat. 4. It will hurt, but we have medicine to help you feel better.

4. Truthful but simple explanations will minimize distorted fears and reduce anxiety.

After receiving report, the nurse is making out assignments. Which of the following clients would be appropriate to assign to unlicensed assistive personnel? 1. A 6-year-old with a femur fracture and a fever. 2. a 13-year-old adolescent with fluctuating vital signs and a new central line. 3. A 7-year-old transferred from the cardiac intensive care unit. 4. An 8-month-old with pneumonia who will be discharged today.

4. Unlicensed assistive personnel can care for a client with pneumonia who will be discharged.

A nurse is teaching new parents about normal development. Voluntary grasp is usually present at what age?

5 months

A 3-month-old infant weighing 10 lb 15 oz has an axillary temperature of 98.9° F. What caloric amount does this child need?

600 calories/day Rationale:An infant requires 108 calories/kg/day. The first step is to change 10 lb 15 oz to 10.9 lb. Then convert pounds to kilograms by dividing pounds by 2.2, which is 10.9/2.2 = 4.954 kg, rounded to 5 kg. The second step is to multiply 108 calories/kg/day (108 × 5 = 540 calories/day). However, this infant requires 10% more calories because of the 1° F temperature elevation. Ten percent of 540 (calories/day) is 54 and 540 + 54 = 594. This infant will require approximately 600 calories/day

When does the child sit unsupported?

8 months

As part of the physical assessment of children, the nurse observes and palpates the fontanels. Which child's fontanel finding should be reported to the healthcare provider?

A 6-month-old with failure to thrive that has a closed anterior fontanel.

Surgery is being delayed for an infant with undescended testes. In collaboration with the healthcare provider and the family, which prescription should the nurse anticipate

A trial of human chorionic gonadotrophic hormone

13. Which procedure uses high-frequency sound waves created by a transducer to produce an image of cardiac structures? A. Echocardiography B. Electrocardiography C. Cardiac catheterization D. Electrophysiology (EPS)

A. Echocardiography Echocardiography uses high-frequency sound waves. The child must lie completely still. With the improvements in technology, diagnosis can sometimes be made without cardiac catheterization. B. Electrocardiography Electrocardiography is a tracing of the electrical path of the depolarization action of myocardial cells. C. Cardiac catheterization Cardiac catheterization is an invasive procedure in which a catheter is threaded into the heart. D. Electrophysiology (EPS) EPS is an invasive procedure in which catheters with electrodes are used to record the impulses of the heart directly from the conduction system.

What is the Gowers sign?

Gowers sign is an indicator of muscular dystrophy; to stand, the child has to "walk" hands up legs

20. As part of the treatment for congestive heart failure, a child is taking the diuretic furosemide (Lasix). As part of the discharge teaching plan the nurse explains that furosemide (Lasix) functions as: A. a diuretic. B. a beta blocker. C. an ACE inhibitor. D. a form of digitalis.

A. a diuretic. Furosemide (Lasix) is a diuretic used to eliminate excess water and salt to prevent reaccumulation of the fluid. B. a beta blocker. Furosemide (Lasix) is a diuretic. C. an ACE inhibitor. Furosemide (Lasix) is a diuretic. D. a form of digitalis. Furosemide (Lasix) is a diuretic.

6. Asthma is now classified into four categories: mild intermittent, mild persistent, moderate persistent, and severe persistent. Clinical features used to differentiate these categories include (select all that apply): A. lung function. B. age of the child C. associated allergies. D. frequency of symptoms. E. frequency and severity of exacerbations.

A. lung function. D. frequency of symptoms. E. frequency and severity of exacerbations.

10. Cystic fibrosis may affect single or multiple systems of the body. The primary factor responsible for possible multiple clinical manifestations is: A. mechanical obstruction caused by increased viscosity of mucous gland secretions. B. atrophic changes in mucosal wall of intestines. C. hypoactivity of the autonomic nervous system. D. hyperactivity of sweat glands.

A. mechanical obstruction caused by increased viscosity of mucous gland secretions. Children with cystic fibrosis have thick mucous gland secretions. The viscous secretions obstruct small passages in organs such as the pancreas. B. atrophic changes in mucosal wall of intestines. Thick mucous secretions are the probable cause of the multiple body system involvement. C. hypoactivity of the autonomic nervous system. There is an identified autonomic nervous system anomaly, but it is not hypoactivity. D. hyperactivity of sweat glands. The sweat glands are not hyperactive. The child loses a greater amount of salt because of abnormal chloride movement.

11. The parent of a child with cystic fibrosis calls the clinic nurse and describes signs and symptoms of tachypnea, tachycardia, dyspnea, pallor, and cyanosis. The most appropriate action by the nurse is to tell the parent to immediately bring the child to the clinic. These symptoms are suggestive of: A. pneumothorax. B. bronchodilation. C. carbon dioxide retention. D. increased viscosity of sputum.

A. pneumothorax. The child is exhibiting signs of increasing respiratory distress suggestive of pneumothorax. The child needs to be seen as soon as possible. B. bronchodilation. These conditions would not produce the symptoms listed. C. carbon dioxide retention. These conditions would not produce the symptoms listed. D. increased viscosity of sputum. The increased viscosity of sputum is characteristic of cystic fibrosis. The described change in respiratory status is potentially due to a pneumothorax.

5. An infant with a congenital heart defect is receiving palivizumab (Synagis). The purpose of this is to: A. prevent RSV infection. B. prevent secondary bacterial infection. C. decrease toxicity of antiviral agents. D. make isolation of infant with RSV unnecessary.

A. prevent RSV infection. Synagis is a monoclonal antibody specific for RSV. Monthly administration is initiated to prevent infection with RSV. B. prevent secondary bacterial infection. The antibody is specific to RSV, not bacterial infection. C. decrease toxicity of antiviral agents. Synagis will have no effect on the toxicity of antiviral agents. D. make isolation of infant with RSV unnecessary. The goal of this drug is prevention of RSV. It will not affect the need to isolate the child if RSV develops.

8. A 4-year-old child needs to use a metered-dose inhaler to treat asthma. The child cannot coordinate her breathing to use it effectively. The appropriate intervention by the nurse is to use a: A. spacer. B. nebulizer. C. peak expiratory flow meter. D. trial of chest physiotherapy.

A. spacer. The medication in a metered-dose inhaler is sprayed into the spacer. The child can then inhale the medication without having to coordinate the spraying and breathing. B. nebulizer. A nebulizer is a mechanism used to administer medications, but it cannot be used with metered-dose inhalers. C. peak expiratory flow meter. This is a measure of pulmonary function not related to medication administration. D. trial of chest physiotherapy. This is unrelated to medication administration.

19. An early sign of congestive heart failure is: A. tachypnea. B. bradycardia. C. inability to sweat. D. increased urine output.

A. tachypnea. Tachypnea is one of the early signs that should be identified. Tachycardia at rest, dyspnea, retractions, and activity intolerance are other physical signs and symptoms. B. bradycardia. Tachycardia, not bradycardia, is one of the symptoms suggestive of congestive heart failure. C. inability to sweat. The child may be diaphoretic. D. increased urine output. Urine output usually will be decreased.

compare the signs and symptoms of acute glomerulonephritis (AGN) with those of nephrosis

AGN: gross hematuria, recent strep infection, hypertension and mild edema Hephrosis: severe edema, massive protinuria, frothy-appearing urine, anorexia

Compare the dietary interventions for AGN and nephrosis

AGN: low sodium diet nephrosis: high protein, low salt diet

Vent septal defect

Abnormal opening between ventricles Acyanotic

At 8 a.m. the unlicensed assistive personnel (UAP) informs the charge nurse that a female adolescent client with acute glomerulonephritis has a blood pressure of 210/110. The 4 a.m. blood pressure reading was 170/88. The client reports to the UAP that she is upset because her boyfriend did not visit last night. What action should the nurse take first?

Administer PRN prescription of nifedipine (Procardia) sublingually.

The nurse is teaching the parents of a 5-year-old with cystic fibrosis about respiratory treatments. Which statement indicates to the nurse that the parents understand?

Administer aerosol therapy followed by postural drainage before meals.

Which intervention(s) should the nurse include in the teaching plan for the mother of a 6-year-old who is experiencing encopresis secondary to a fecal impaction? (Select all that apply.)

Administer mineral oil daily. Eliminate dairy products. Initiate consistent toileting routine. Rationale: Encopresis is fecal incontinence, usually as the result of recurring fecal impaction and an enlarged rectum caused by chronic constipation. Encopresis is managed through bowel retraining with mineral oil, eliminating dairy products, and initiating a regular toileting routine. A high-fiber diet and increased daily fluids are components of care for a child with encopresis.

A 16-year-old is brought to the Emergency Center with a crushed leg after falling off a horse. The adolescent's last tetanus toxoid booster was received eight years ago. What action should the nurse take?

Administer tetanus toxoid booster.

3yo is hospitalized for femur fracture. As her nurse, what nursing action would help foster the child's sense of autonomy?

Allow the child to administer her own Keflex (cehphalexin) via oral syringe.

The nurse should teach the parents of a child with a cyanotic heart defect to perform which action when a hypercyanotic spell occurs?

Allow the child to assume a knee-chest position, with the head and chest slightly elevated. Rationale: Assuming a knee-chest position with the head and chest slightly elevated will help restore hemodynamic equilibrium.

Which nursing intervention(s) is (are) therapeutic when caring for a hospitalized toddler? (Select all that apply.)

Allow the toddler to choose a colored Band-Aid after an injection & Give brief but simple explanations to the child before procedures. Rationale: Giving the toddler a choice may increase autonomy in the hospitalized setting. Brief but simple explanations are beneficial with the toddler. Separation from the parent can cause emotional distress. Regression is expected, and bedwetting is not an indication for a urinary catheter. The nurse should encourage age-appropriate toys to be brought in from home.

Which nursing diagnosis has the highest priority when planning care for an infant with eczema?

Altered comfort (pruritus) related to vesicular skin eruptions Rationale: Altered comfort (pruritus) has the highest priority because itching will cause the infant to scratch, creating complications such as scarring or infection.

Which of the following should the nurse expect to note as a frequent complication for a child with congenitalheart disease? a.Susceptibility to respiratory infection b. Bleeding tendencies c. Frequent vomiting and diarrhea d. Seizure disorder

Answer A Children with congenital heart disease are more prone to respiratory infections.Bleeding tendencies, frequent vomiting, and diarrhea and seizure disorders are not associated with congenital heart disease

A newborn's failure to pass meconium within the first 24 hours after birth may indicate which of the following? a.Hirschsprung disease b. Celiac disease c. Intussusception d. Abdominal wall defect

Answer A Failure to pass meconium within the first 24 hours after birth may be an indication of Hirschsprung disease, a congenital anomaly resulting in mechanical obstruction due to inadequate motility in an intestinal segment. Failure to pass meconium is not associated with celiac disease, intussusception, or abdominal wall defect.

Which of the following should the nurse do first after noting that a child with Hirschsprung disease has a fever and watery explosive diarrhea? a. Notify the physician immediately b. Administer antidiarrheal medications c. Monitor child ever 30 minutes d. Nothing, this is characteristic of Hirschsprung disease

Answer A For the child with Hirschsprung disease, fever and explosive diarrhea indicate enterocolitis, a life-threatening situation. Therefore, the physician should be notified immediately. Generally, because of the intestinal obstruction and inadequate propulsive intestinal movement, antidiarrheals are not used to treat Hirschsprung disease. The child is acutely ill and requires intervention, with monitoring more frequently than every 30 minutes.Hirschsprung disease typically presents with chronic constipation.

Which of the following parameters would the nurse monitor to evaluate the effectiveness of thickened feedings for an infant with gastroesophageal reflux (GER)? a.Vomiting b. Stools c. Uterine d. Weight

Answer A Thickened feedings are used with GER to stopthe vomiting. Therefore, the nurse wouldmonitor the child's vomiting to evaluate theeffectiveness of using the thickened feedings.No relationship exists between feedings andcharacteristics of stools and uterine. Iffeedings are ineffective, this should be notedbefore there is any change in the child's weight.

While assessing a newborn with cleft lip,the nurse would be alert that which of the following will most likely be compromised? a.Sucking ability b. Respiratory status c. Locomotion d. GI function

Answer A. Because of the defect, the child will be unable to from the mouth adequately around nipple, there by requiring special devices to allow for feeding andsucking gratification. Respiratory status may be compromised if the child is fed improperly or during postoperative period, Locomotion would be a problem for the older infant because of the use of restraints. GI functioning is not compromised in the child with a cleft lip

A child with leukemia is being discharged after beginning chemotherapy. Which of the following instructions will the nurse include when teaching the parents of this child? a)provide a diet low in protein and high carbohydrates b) avoid fresh vegetables that are not cooked or peeled c) notify the doctor if the child's temperature exceeds 101 F (39C) d) increase the use of humidifiers throughout the house

Answer B fresh fruits and vegetables harbor microorganisms, which can cause infections in immune-compromised child. Fruits and vegetables should either be peeled or cooked. The physician should be notified of a temperature above 100F, a diet low in protein is not indicated, and humidifiers harbor fungi in the water containers.

Which of the following health teachings regarding sickle cell crisis should be included by the nurse? a) it results from altered metabolism and dehydration b) tissue hypoxia and vascular occlusion cause the primary problems c) increased bilirubin levels will cause hypertension d) there are decreased clotting factors with an increase in white blood cells

Answer B tissue hypoxia occurs as a result of the decreased oxygen-carrying capacity of the red blood cells. The sickled cells begin to clump together, which leads to vascular occlusion.

When providing postoperative care for the child with a cleft palate, the nurse should position the child in which of the following positions? a.Supine b. Prone c. In an infant seat d. On the side

Answer B. Postoperatively children with cleft palate should be placed on their abdomens to facilitate drainage.If the child is placed in the supine position, he or she may aspirate. Using an infant seat does not facilitate drainage. Side-lying does not facilitate drainage as well as the prone position

When assessing a child for possible intussusception, which of the following would be least likely to provide valuable information? a.Stool inspection b. Pain pattern c. Family history d. Abdominal palpation

Answer C Because intussusception is not believed to have a familial tendency, obtaining a family history would provide the least amount of information. Stool inspection, pain pattern, and abdominal palpation would reveal possible indicators of intussusception. Current, jelly-like stools containing blood and mucus are an indication of intussusception. Acute,episodic abdominal pain is characteristics of intussusception. A sausage-shaped mass may be palpated in the right upper quadrant.

A client and her husband are positive for the sickle cell trait. The client asks the nurse about chances of her children having sickle cell disease. Which of the following is appropriate response by the nurse? a)one of her children will have sickle cell disease b) only the male children will be affected c) each pregnancy carries a 25% chance of the child being affected d) if she had four children, one of them would have the disease

Answer C In autosomal recessive traits, both parents are carriers. There is a 25% chance with each pregnancy that a child will have the disease.

A 16 month old child diagnosed with Kawasaki Disease (KD) is very irritable,refuses to eat, and exhibits peeling skin on the hands and feet. Which of the following would the nurse interpret as the priority? a)applying lotions to the hands and feet b) offering foods the toddler likes c) placing the toddler in a quiet environment d) encouraging the parents to get some rest

Answer C One of the characteristics of children with KD is irritability. They are often inconsolable.Placing the child in a quiet environment may help quiet the child and reduce the workload of the heart. The child's irritability takes priority over peeling of the skin.

After teaching the parents of a preschooler who has undergone T and A(Tonsillectomy and Adenoidectomy) about appropriate foods to give the child afterdischarge, which of the following, if stated by the parents as appropriate foods, indicates successful teaching? a)meatloaf and uncooked carrots b) pork and noodle casserole c) cream of chicken soup and orange sherbet d) hot dog and potato chips

Answer C for the first few days after a T and A (Tonsillectomy and Adenoidectomy) , liquids and soft foods are best tolerated by the child while the throat is sore. Avoid hard and scratchy foods until throat is healed.

An 8 year old child has been diagnosed to have iron deficiency anemia. Which ofthe following activities is most appropriate for the child to decrease oxygen demands on the body? a)Dancing b) playing video games c) reading a book d) riding a bicycle

Answer C reading a book is restful activity and can keep the child from becoming bored. Choices a, b, and d require too much energy for a child with anemia and can increase oxygen demands on the body.

Which of the following nursing diagnoses would be inappropriate for the infant with gastroesophageal reflux(GER)? a.Fluid volume deficit b. Risk for aspiration c. Altered nutrition: less than body requirements d. Altered oral mucous membranes

Answer D GER is the backflow of gastric contents into the esophagus resulting from relaxation or incompetence of the lower esophageal (cardiac)sphincter. No alteration in the oral mucous membranes occurs with this disorder. Fluid volume deficit, risk for aspiration, and altered nutrition are appropriate nursing diagnoses

A client with hemophilia has a very swollen knee after falling from bicycleriding. Which of the following is the first nursing action? a)initiate an IV site to begin administration of cryoprecipitate b) type and cross-match for possible transfusion c) monitor the client's vital signs for the first 5 minutes d) apply ice pack and compression dressings to the knee

Answer D rest, ice, compression, and elevation (RICE)are the immediate treatments to reduce the swelling and bleeding into the joint. These are the priority actions for bleeding into the joint of a client with hemophilia

A child diagnosed with tetralogy of fallot becomes upset, crying and thrashing around when a blood specimen is obtained.The child's color becomes blue and respiratory rate increases to 44 bpm.Which of the following actions would the nurse do first? a)obtain an order for sedation for the child b) assess for an irregular heart rate and rhythm c) explain to the child that it will only hurt for a short time d) place the child in knee-to-chest position

Answer D the child is experiencing a "tet spell" or hypoxic episode.Therefore the nurse should place the child in a knee-to-chest position. Flexing the legs reduces venous flow of blood from lower extremities and reduces the volume of blood being shunted through the interventricular septal defect and the overriding aorta in the child with tetralogy of fallot. As a result, the blood then entering the systemic circulation has higher oxygen content, and dyspnea is reduced. Flexing the legs also increases vascular resistance and pressure in the left ventricle. An infant often assumes a knee-to-chest position to relieve dyspnea. If this position is ineffective, then the child may need sedative. Once the child is in this position, the nurse may assess for an irregular heart rate and rhythm.Explaining to the child that it will only hurt for a short time does nothing to alleviate hypoxia.

Which of the following would the nurse perform to help alleviate a child's joint pain associated with rheumatic fever? a)maintaining the joints in an extended position b) applying gentle traction to the child's affected joints c) supporting proper alignment with rolled pillows d) using a bed cradle to avoid the weight of bed lines on the joints

Answer D for a child with arthritis associated with rheumatic fever, the joints are usually so tender that even the weight of bed linens can cause pain. Use of the bed cradle is recommended to help remove the weight of the linens on painful joints. Joints need to be maintained in good alignment, not positioned inextension, to ensure that they remain functional.Applying gentle traction to the joints is not recommended because traction is usually used to relieve muscle spasms, not typically associated with rheumatic fever. Supporting the body in good alignment and changing the client's position are recommended, but these measures are not likely to relieve pain.

A hospitalized 16 yr old male refuses all visits fr. his classmates because he is concerned about his distorted appearance. To increase the clients social interaction, what intervention is best for the nurse to initiate?

Arrange for an internet connection in the client's room for email communication

A 6-month-old infant w/ congestive heart failure (CHF) is receiving digoxin elixir. Which observation by the nurse warrants immediate intervention?

Apical heart rate of 60

A 2-year-old child recently diagnosed with hemophilia A is discharged home. What information should the nurse include in a teaching plan about home care?

Apply pressure and ice for bleeding while elevating and resting the extremity.

During routine screening at a school clinic, an otoscope examination of a child's ear reveals a tympanic membrane that is pearly gray, slightly bulging, and not movable. Based on these findings, what action should the nurse take?

Ask if the child has had a cold, runny nose, or any ear pain lately. Rationale: The tympanic membrane is normally pearly gray, not bulging, and moves when a client blows against resistance or when a small puff of air is blown into the ear canal. Because these findings are not completely normal, further assessment of history and related signs and symptoms are needed to interpret the findings accurately.

How would the nurse conduct a scoliosis screening?

Ask the child to bend forward form the hips, with arms hanging free. Examine the child for a curve in the spine, a rib hump, and hip asymmetry

The nurse is conducting an initial admission assessment of a 12-month-old child in celiac crisis. Which intervention is most important for the nurse to implement?

Assess the child's mucous membranes and skin turgor Rationale: An infant having a celiac crisis has severe diarrhea and is at high risk for fluid volume deficit. The nurse should first assess for indications of fluid volume deficit

A child is rescued from a burning house and brought to the emergency room with partial-thickness burns on the face and chest. Which action should the nurse implemented first?

Assess the child's respiratory status

"A mother tells the nurse that her child does not want anything to do with toilet training and yells "NO!" consistently when she tries to toilet train. The child is 2 years old. According to Erikson, the nurse interprets that the child is experiencing which psychosocial crisis?" a. autonomy vs shame and doubt b. initiative vs guilt c. industry vs inferiority d. trust vs mistrust

Autonomy vs shame and doubt

Cystic fibrosis

Autosomal recessive Absence of pancreatic enzymes Greasy stool Chronic pulmonary infections High sweat test Pancreatic enzymes with meals

1. Apnea of infancy has been diagnosed in an infant who will soon be discharged with home monitoring. Part of the discharge teaching by the nurse for the parents includes: A. administration of IV fluids. B. cardiopulmonary resuscitation. C. reassurance that the infant cannot be electrocuted during monitoring. D. advisement that the infant not be left with other caretakers, such as babysitters.

B. cardiopulmonary resuscitation. CPR is essential for parents and caregivers to know. A. administration of IV fluids. Most likely, the child will not have venous access, so home IV therapy is not necessary. C. reassurance that the infant cannot be electrocuted during monitoring. The monitor is insulated and grounded. D. advisement that the infant not be left with other caretakers, such as babysitters. The parents should arrange for other caregivers to help out. All need to be taught how to use the monitoring equipment and how to perform CPR.

21. The nurse is preparing to administer digoxin (Lanoxin) orally to a 9-month-old infant. The nurse checks the dose and prepares to draw up 4 ml of the drug. The most appropriate nursing action is to: A. mix the dose with juice to disguise the drug's taste. B. hold the dose because a dosage error is suspected, check the orders, and check the dosage with another nurse. C. check the heart rate, then administer the dose by placing it at the back and side of the mouth. D. check the heart rate, then administer the dose by letting the infant suck it through a nipple.

B. hold the dose because a dosage error is suspected, check the orders, and check the dosage with another nurse. Digoxin is often prescribed in micrograms. Rarely is more than 1 ml administered to an infant. Because it is a potentially dangerous drug, administration guidelines are very precise. Pediatric medication dosages should be checked with another professional before administration. A. mix the dose with juice to disguise the drug's taste. The nurse has drawn up too much medication. C. check the heart rate, then administer the dose by placing it at the back and side of the mouth. Checking the heart rate and administering the dose by placing it at the back and side of the mouth is a correct procedure, but too much medication has been prepared. D. check the heart rate, then administer the dose by letting the infant suck it through a nipple. Checking the heart rate and administering the dose by letting the infant suck it through a nipple is a correct procedure, but too much medication has been prepared.

The nurse observes a 4-year-old boy in a day care setting. Which behavior should the nurse expect this child to exhibit?

Boasts aggressively when telling a story Rationale: Four-year-old children are aggressive in their behavior and enjoy telling tales

4. The mother of a 20-month-old child tells the nurse that the child has a barking cough at night. The child's temperature is 37° C (98.6° F). Based on the nurse's knowledge of upper respiratory infections, this is a symptom of croup. The nurse should recommend to: A. control the fever with acetaminophen and call if the cough gets worse tonight. B. try a cool-mist vaporizer at night and watch for signs of difficulty breathing. C. try over-the-counter cough medicine and come to the clinic tomorrow if there is no improvement. D. admit to the hospital and observe for impending epiglottitis.

B. try a cool-mist vaporizer at night and watch for signs of difficulty breathing. Because the child is not having difficulty breathing, the nurse should teach the parents the signs of respiratory distress and tell them to come to the emergency department if they develop. Cool mist is recommended to provide relief. A. control the fever with acetaminophen and call if the cough gets worse tonight. The child does not have a temperature to manage. C. try over-the-counter cough medicine and come to the clinic tomorrow if there is no improvement. Cough suppressants are not indicated. D. admit to the hospital and observe for impending epiglottitis. A barking cough and temperature of 37° C are characteristic of laryngotracheobronchitis, not epiglottitis.

The nurse is preparing a teaching plan for the mother of a child who has been diagnosed with celiac disease. Choosing which lunch will be within the therapeutic management of a child with celiac disease?

Baked chicken, coleslaw, soda, and frozen fruit dessert Rationale: A child with celiac disease is managed on a gluten-free diet, which eliminates food products containing oats, wheat, rye, or barley

What antecedent event occurs with AGN?

Beta-hemolytic streptococcal infection

Norms for growth and development

Birth weight doubles by 6 months, triples by yr Peek a boo by 6 month Sits upright without support by 8 pincher grip by 10-12

A child is admitted to the hospital for confirmation of a diagnosis of acute lymphoblastic leukemia. During the initial nursing assessment, which symptoms will this child most likely exhibit?

Bone pain, pallor Rationale: Leukemic cells invade the bone marrow, gradually causing a weakening of the bone and a tendency toward pathologic fractures. As leukemic cells invade the periosteum, increasing pressure causes severe pain and anemia results from decreased erythrocytes, causing pallor.

Autosomal recessive

Both parents must be heterozygous, or carriers of the recessive trait, for the disease to be expressed in their offspring. With each pregnancy, there is a 1:4 chance of the infant having the disease. However, all children of such parents CAN get the disease - NOT 25% of them.

Resp failure infants

Brady Deceased rr Apneic Deceased loc Grunting Deceased breath sounds

A 3-week-old infant is referred to an orthopedic clinic because the pediatrician heard a click when flexing the child's right hip during a routine physical examination. The orthopedic physician suspects that the child might have developmental dysplasia of the hip (DDH). The parents ask the nurse to identify risk factors commonly associated with DDH. Which response is accurate?

Breech presentation

A mother calls the clinic because her 6-year-old son, who has been taking prescribed antibiotics for 7 of the previous 10 days, continues to have a cough that she reports is worsening. Further questioning by the nurse reveals that the cough is nonproductive. What advice should the nurse provide to this mother?

Bring the child to the clinic today for an examination related to the cough. Rationale: The child should be evaluated as soon as possible for pneumonia. Antibiotics usually improve symptoms during the first few days of treatment but should be continued for the full prescribed course. A continued cough after 7 days of antibiotic treatment may indicate an infectious process in the lower lungs, which could cause a nonproductive cough. Children with pneumonia can deteriorate unexpectedly and rapidly and can become seriously ill, with no sputum production

When does birth length double?

By 4 years

12. Absorption of fat-soluble vitamins is decreased in children with cystic fibrosis; therefore supplementation of which vitamins is necessary? A. C, D B. A, E, K C. A, D, E, K D. C, folic acid

C. A, D, E, K A, D, E, and K are the fat-soluble vitamins that need to be supplemented. A. C, D C is not one of the fat-soluble vitamins. B. A, E, K D also needs to be supplemented. D. C, folic acid C and folic acid are not fat-soluble.

23. The primary therapy for secondary hypertension in children is: A. eating a low-salt diet. B. reducing body weight. C. treating the underlying cause. D. increasing exercise and fitness.

C. treating the underlying cause. Secondary hypertension is a result of an underlying disease process or structural abnormality. It is usually necessary to treat the problem before the hypertension will be controlled. A. eating a low-salt diet. The low-salt diet, weight reduction, and increased exercise and fitness are usually effective for essential hypertension. B. reducing body weight. The low-salt diet, weight reduction, and increased exercise and fitness are usually effective for essential hypertension. D. increasing exercise and fitness. The low-salt diet, weight reduction, and increased exercise and fitness are usually effective for essential hypertension.

What are the metabolic effects of PKU?

CNS damage, mental retardation and decreased melanin

A 4-year-old boy was admitted to the emergency room with a fractured right ulna and a short arm cast is applied. When preparing the parents to take the child home, which discharge instruction has the highest priority?

Call the healthcare provider immediately if his nail beds appear blue.

Directional trends in growth and development are easily seen in the neonate. Which term describes development in the head-to-tail direction?

Cephalocaudal trend

When taking the health history of a child, the nurse knows that which finding is an early indication of hypothyroidism in children?

Cessation of growth in a child that had been normal.

A 4-year-old girl continues to interrupt her mother during a routine clinic visit. The mother appears irritated with the child and asks the nurse, "Is this normal behavior for a child this age?" The nurse's response should be based on which information?

Children need to retain a sense of initiative without impinging on the rights and privileges of others.

A full term infant is admitted to the newborn nursery. After careful assessment, the nurse suspects that the infant may have an esophageal atresia. Which symptoms are this newborn likely to exhibit?

Choking, coughing, and cyanosis.

A nurse is preparing to end the shift and receives a lab report stating that a child with asthma has a theophylline level of 15 mcg/dL. Which action should the nurse take?

Communicate the result to the oncoming nurse and document. Rationale: The therapeutic level of theophylline is 10 to 20 mcg/dL, so the child's level is within the therapeutic range. This information evaluates the prescribed therapy and should be communicated in the nurse's report.

A 2-year-old child with Down syndrome is brought to the clinic for his regular physical examination. The nurse knows which problem is frequently associated with Down syndrome?

Congenital heart disease.

The nurse is planning postoperative care for a child who has had a cleft lip repair. What is the most important reason to minimize this child's crying during the recovery period?

Crying stresses the suture line. Rationale: Prevention of stress on the lip suture line is essential for optimum healing and the cosmetic appearance of a cleft lip repair.

In making the initial assessment of a 2-hour-old infant, which finding should lead the nurse to suspect a congenital heart defect?

Diminished femoral pulses Rationale: Diminished femoral pulses (D) could indicate coarctation of the aorta.

7. A child with asthma is having pulmonary function tests. Which explains the purpose of the forced expiratory volume (FEV1)? A. Confirm the diagnosis of asthma B. Determine the cause of asthma C. Identify the "triggers" of asthma D. Assess the severity of asthma

D. Assess the severity of asthma The forced expiratory volume measures the maximum amount of air that can be forcefully exhaled in the first second. This can provide an objective measure of pulmonary function compared with the child's baseline. A. Confirm the diagnosis of asthma Diagnosis of asthma is made on the basis of clinical manifestations, history, and physical examination. B. Determine the cause of asthma The cause of asthma is inflammation, bronchospasm, and obstruction. C. Identify the "triggers" of asthma Some of the triggers of asthma are identified with allergy testing.

16. The beneficial effect of performing surgery for patent ductus arteriosus (PDA) is to prevent which complication? A. Pulmonary infection B. Right-to-left shunt of blood C. Decreased workload on left side of heart D. Increased pulmonary vascular congestion

D. Increased pulmonary vascular congestion A patent ductus arteriosus (PDA) allows blood to flow from the aorta (high pressure) to the pulmonary artery (low pressure). If the PDA stays open, increased pulmonary congestion can occur. A. Pulmonary infection The increased pulmonary vascular congestion is the primary complication. B. Right-to-left shunt of blood The blood is shunted left to right. C. Decreased workload on left side of heart The increased pulmonary vascular congestion is the primary complication.

18. Which is classified as a mixed-blood cardiac defect? A. Pulmonic stenosis B. Atrial septal defect C. Patent ductus arteriosus D. Transposition of the great arteries

D. Transposition of the great arteries Transposition of the great arteries allows the mixing of blood in the heart. A. Pulmonic stenosis Pulmonic stenosis is classified as an obstructive defect. B. Atrial septal defect Atrial septal defect is classified as a defect with increased pulmonary blood flow. C. Patent ductus arteriosus Patent ductus arteriosus is classified as a defect with increased pulmonary blood flow.

3. A child with acute streptococcal pharyngitis should be treated with antibiotics to prevent: A. otitis media. B. diabetes insipidus. C. nephrotic syndrome. D. acute rheumatic fever.

D. acute rheumatic fever. Children with group A beta-hemolytic streptococci (GABHS) infection are at risk for acute rheumatic fever and acute glomerulonephritis. A. otitis media. Otitis media is not a sequela to GABHS. B. diabetes insipidus. Diabetes insipidus is not a sequela to GABHS. C. nephrotic syndrome. Children are at risk for glomerulonephritis, not nephrotic syndrome.

17. A young child with tetralogy of Fallot may assume a posturing position as a compensatory mechanism. The position automatically assumed by the child is: A. Low Fowler's. B. prone. C. supine. D. knee-chest.

D. knee-chest. The squatting or knee-chest position decreases the amount of blood returning to the heart and allows the child time to compensate. A. Low Fowler's. Low Fowler's position would assist with respiratory issues but would not assist with the need for cardiac compensation. B. prone. Prone does not offer any advantage to the child. C. supine. Supine does not offer any advantage to the child.

22. Nursing care of the infant or child with congestive heart failure includes: A. forcing fluids appropriate for the patient's age. B. monitoring respirations during active periods. C. giving larger feedings less often to conserve energy. D. organizing activities to allow for uninterrupted sleep.

D. organizing activities to allow for uninterrupted sleep. The child needs to be well rested before feeding. The child's needs should be met as quickly as possible to minimize crying. The nurse must organize care to facilitate a decrease in the child's energy expenditure. A. forcing fluids appropriate for the patient's age. The child who has congestive heart failure has an excess of fluid. B. monitoring respirations during active periods. Monitoring of vital signs is appropriate, but minimizing energy expenditure is a priority. C. giving larger feedings less often to conserve energy. The child often cannot tolerate larger feedings.

15. Traditionally, congenital heart defects have been divided into acyanotic or cyanotic defects. Based on the nurse's knowledge of congenital heart defects, this system is: A. helpful, because it explains the hemodynamics involved. B. helpful, because cyanotic defects are easily identified. C. problematic, because cyanosis is rarely present in children. D. problematic, because acyanotic heart defects may have cyanosis.

D. problematic, because acyanotic heart defects may have cyanosis. The classification is problematic. Children with traditionally named acyanotic defects may be cyanotic, and children with traditionally classified cyanotic defects may appear pink, although they may eventually become cyanotic. A. helpful, because it explains the hemodynamics involved. The classification does not reflect the path of blood flow within the heart. B. helpful, because cyanotic defects are easily identified. Children with cyanosis may be easily identified, but that does not help with the diagnosis. C. problematic, because cyanosis is rarely present in children. Cyanosis is present when children have defects in which oxygenated blood and unoxygenated blood are mixed.

14. A nurse caring for a child post cardiac catheterization assesses that the distal pulse of the catheter site is weaker and capillary refill is less than three seconds. The most appropriate nursing action is to: A. elevate the affected extremity. B. notify the physician of the observation. C. apply warm compresses to the insertion site. D. record the assessment finding and continue to monitor.

D. record the assessment finding and continue to monitor. The pulse distal to the catheter insertion site may be weaker for the first few hours after catheterization. It should gradually increase in strength. A. elevate the affected extremity. Elevation is not necessary; the extremity is kept straight. B. notify the physician of the observation. Because a weaker pulse is an expected finding, the nurse should document this and continue to monitor. C. apply warm compresses to the insertion site. The insertion site is kept dry.

2. A humidified atmosphere is recommended for a young child with an upper respiratory tract infection because this environment facilitates: A. liquefying secretions. B. improving oxygenation. C. promoting ventilation. D. soothing inflamed mucous membrane.

D. soothing inflamed mucous membrane. By humidifying the inspired air, the membranes inflamed by the infection and dry air are soothed. A. liquefying secretions. The size of the droplets is too large to liquefy secretions. B. improving oxygenation. No additional oxygen is provided with humidified air. C. promoting ventilation. The humidity has no effect on ventilation.

9. One of the goals for children with asthma is to prevent respiratory infection. The reason for this goal is that respiratory infection can: A. encourage exercise-induced asthma. B. increase sensitivity to allergens. C. lessen the effectiveness of medications. D. trigger an episode or aggravate an asthmatic state.

D. trigger an episode or aggravate an asthmatic state. Respiratory infections can trigger an asthmatic attack. Annual influenza vaccine is recommended. All respiratory equipment should be kept clean. A. encourage exercise-induced asthma. Exercise-induced asthma is caused by vigorous activity. B. increase sensitivity to allergens. Sensitivity to allergens is independent of respiratory infection. C. lessen the effectiveness of medications. The infection affects the asthma, not the medications.

A 3-year-old boy is brought to the emergency room because he swallowed an entire bottle of children's vitamin pills. Which intervention should the nurse implement first?

Determine the child's pulse and respirations.

The nurse expects a 2-year-old child to exhibit which behavior?

Display possessiveness with toys. Rationale: Two-year-old children are egocentric and unable to share with other children.

The mother of a preschool-aged child asks the nurse if it is all right to administer Pepto Bismol to her son when he "has a tummy ache." After reminding the mother to check the label of all over-the-counter drugs for the presence of aspirin, which instruction should the nurse include when replying to this mother's question?

Do not give if the child has chickenpox, the flu, or any other viral illness.

School age

Each yr gains 4-6 lbs grows 2 inches Socialization with peers very important

Which restraint should be used for a toddler after a cleft palate repair?

Elbow

The mother of a 4-year-old child asks the nurse what she can do to help her other children cope with their sibling's hospitalizations. Which is the best response that the nurse should offer?

Encourage the mother to have the children visit the hospitalized sibling.

A 16yo male is hospitalized for cystic fibrosis. He will be an inpatient for 2 weeks while he receives IV antibiotics. As the nurse caring for this pt. what action can you take that will most enhance his psychosocial development?

Encourage the teen's friends to visit him in the hospital.

The nurse is having difficulty communicating with a hospitalized 6-year-old child. Which approach by the nurse is most helpful in establishing communication?

Engage the child through drawing pictures.

The nurse is teaching the parents of a 2-year-old child with a congenital heart defect about signs and symptoms of congestive heart failure. Which information about the child is most important for the parents to report to the health care provider?

Exhibits a sudden and unexplained weight gain Rationale: Sudden and unexplained weight gain (B) can indicate fluid retention and is a sign of congestive heart failure.

what are the physical assessment findings for a child with asthma?

Expiratory wheezing, rales, tight cough, and signs of altered blood gases

A 6-year-old is admitted to the pediatric unit after falling off a bicycle. Which intervention should the nurse implement to assist the child's adjustment to hospitalization?

Explain hospital schedules to the child, such as mealtimes.

A 15-year-old girl tells the school nurse that all of her friends have started their periods and she feels abnormal because she has not. Which response is best for the nurse provide?

Explain that menarche varies and occurs between the ages of 12 and 18 years

The nurse admits a child to the intensive care unit with a diagnosis of acquired aplastic anemia. What is the most common cause of this type of anemia?

Exposure to certain drugs Rationale: Aplastic anemia often follows exposure to certain drugs such as chloramphenicol, sulfonamides, and phenylbutazone (Butazolidin), insecticides such as DDT, and chemicals, especially, benzene.

During discharge teaching of a child with juvenile rheumatoid arthritis, the nurse should stress to the parents the importance of obtaining which diagnostic testing?

Eye exams.

Preschool

Favorite word is why Sentences of 5-8

the nurse must evaluate possible complications associated with traction. Which of the following assessment indicates a potential CIRCULATORY complication? Numbness and tingling of toes---this is NEURO not circulatory Poor dorsiflexion of the foot Capillary refill of 1-2 seconds Foot feels cool to touch

Foot feels cool to touch

Why are fractures of the epiphyseal plate a special concern?

Fractures of the epiphyseal plate (growth plate) may affect the growth of the limb

A 17yo male is being seen in the ER. In order to obtain the adolescent's health information, his nurse should:

Gather info. during a casual conversation

The nurse is developing a plan of care for a 3-year-old who is scheduled for a cardiac catheterization. To assist in decreasing anxiety for the child on the day of the procedure, which intervention is best for the nurse to implement?

Give the child a ride on a gurney to visit the cardiac catheterization lab and meet a nurse who works there.

A 12-month-old boy is admitted with a respiratory infection and possible pneumonia. He is placed in a mist tent with oxygen. Which nursing intervention has the greatest priority for this infant?

Have a bulb syringe readily available to remove secretions

A burned child is brought to the emergency department, and the nurse uses a modified rule of nines to estimate the percentage of the body burned. When calculating the percentage of burn, which parts of the child's body is proportionally larger than an adult's?

Head and neck Rationale: The standard rule of nines is inaccurate for determining burned body surface areas with children because a child's head and neck are proportionately larger than an adult's. Specially designed charts are commonly used to measure the percentage of burn in children.

When admitting a child in the acute phase of glomerulonephritis (AGN), the nurse should expect the urinalysis to show which of the following? Bacteria and hematuria Hematuria and proteinuria Bacteria and increased specific gravity Proteinuria and increased specific gravity

Hematuria and proteinuria

What is the most common postoperative complication following a tonsillectomy? Describe the signs and symptoms of this complication

Hemorrhage frequent swallowing, vomiting fresh blood, and clearing throat

Digoxin therapy in kids

Hold infants <100, kids <80 Don't skip or make up dose Give 1-2 hrs before meals S/s over dose: vomiting, diarrhea, muscle weakness, drowsiness

Patent ductus arterious

Hole between aorta and pulmonary artery Acyanototic So are coarctation of aorta and aortic stenosis

Methylphenidate (Ritalin) is prescribed for a child with attention deficit hyperactivity disorder (ADHD). The nurse provides instructions to the mother regarding the administration of the medication. Which statement by the mother would indicate a need for further instructions? a. I will administer the medication with the noontime meal b. I will keep the medication tightly capped and away from direct heat c. I should inform the school nurse that my child is taking this medication d. I should avoid giving the medications on Saturdays and Sundays to provide a medication holiday

I should avoid giving the medications on Saturdays and Sundays to provide a medication holiday

The nurse is giving preoperative instruction to a 14-year-old female client who is scheduled for surgery to correct a spinal curvature. Which statement by the client best demonstrates that learning has taken place?

I understand that I will be in a body cast and I will show you how you taught me to turn

"Methylphenidate hydrochloride (Ritalin) is prescribed for a 10-year-old child diagnosed with attention deficit hyperactivity disorder, and the nurse provides instructions to the mother about administration of the medication. The nurse determines that the mother understands the instructions when the mother states:" a. I will give my child the medication at bedtime so that he will be rested and alert for school the next day b. I will give my child the medication after meals to obtain the full effects of the medication c. I will give my child the medication at breakfast and lunch to prevent insomnia d. I will give my child the medication with water to prevent kidney damage

I will give my child the medication at breakfast and lunch to prevent insomnia

A nurse caring for a child with congestive heart failure provides instructions to the parents regarding the administration of digoxin (Lanoxin). Which statement by the mother indicates a need for further instructions? a. if my child vomits after I give the medication, I will not repeat the dose b. I will check my child's pulse before giving the medication c. I will check the dose of the medication with my husband before I give the medication d. I will mix the medication with food

I will mix the medication with food

"A nurse is providing instructions to the mother of a child with a diagnosis of strabismus of the left eye, and the nurse reviews the procedure for patching the child. The nurse determines that the mother understands the procedure if the mother makes which statement?" a. I will place the patch on the right eye b. I will place the patch on both eyes c. I will place the patch on the left eye d. I will alternate the patch from the right to the left eye every hour

I will place the patch on the right eye

Resp distress infants

Increase rr axillAry muscles for breathing Restlessness Nasal flaring Tripod Grunting Cyanotic

Age groups concepts of bodily injury

Infants: After 6 months, their cognitive development allows them to remember pain. Toddlers: Fear intrusive procedures. Preschoolers: Fear body mutilation. School age: Fear loss of control of their body. Adolescent: Major concern is change in body image.

A 4-year-old child has cystic fibrosis. Which stage of Erikson's theory of psychosocial development is the nurse addressing when teaching inhalation therapy?

Initiative Rationale: Children 4 to 5 years of age are in the "Initiative vs. Guilt" stage of Erikson's theory of psychosocial development

A 7-month-old infant with a rotavirus causing severe diarrhea is admitted for treatment. Which intervention should the nurse implement first?

Insert an intravenous (IV) line and begin IV fluids. Rationale: An infant with severe diarrhea is at high risk for dehydration, so the nurse's priority is to initiate IV fluids

At which point during the physical examination should a child with asthma be assessed for the presence or absence of intercostal retractions?

Inspiration Rationale: Intercostal retractions result from respiratory effort to draw air into restricted airways. The retractions will not be noticeable when air is expelled from the lungs, such as when the client is coughing or expiring

Following immunization, what teaching should the nurse provide to the parents?

Irritability, fever (<102 degrees F), redness and soreness at injection site for 2-3days are normal side effects of DPT and IPV administration.Call healthcare provider if seizures, high fever, or high -pitched crying occur.A warm washcloth on the thigh injection site and "bicycling" the legs w/each diaper change will decrease soreness.Acetaminophen (Tylenol) is administered orally every 4-6 hours (10-15 mg/Kg)

The nurse is examining a male child experiencing an exacerbation of juvenile rheumatoid arthritis (JRA) and notes that his mobility is greatly reduced. What is the most likely cause of the child's impaired mobility?

Joint inflammation Rationale: Joint inflammation and pain are the typical manifestations of an exacerbation of JRA

What two formulas are prescribed for infants with PKU?

Lofenalac Phenex-1

When caring for a child with congenital heart disease and polycythemia, which nursing intervention has the highest priority?

Maintaining adequate hydration Rationale: The key word in this question is polycythemia. Hydration decreases blood viscosity and the risk for thrombus formation, the most common complication of polycythemia.

What are the outcomes of untreated congenital hypothyroidism?

Mental retardation and growth failure

When inserting a nasogastric tube into the stomach of a 3-month-old infant, which nursing intervention is most important to implement?

Monitor the infant's heart rate

A 3 week old newborn is brought to the clinic for a follow up after a home birth. The mother reports that her child bottle feeds for 5 min only and falls asleep. The nurse auscultates a loud murmur characteristic of a ventricular septal defect (VSD), and finds the newborn is acyanotic w/ respiratory rate of 64 breaths per min. What instruction should the nurse provide the mother to ensure the infant is receiving adequate intake? (Select all that apply)

Monitor the infant's weight and # of wet diapers per day Increase the infant's intake per feeding by 1-2 ounces per week allow the infant to rest and reefed on demand or every 2 hrs use a softer nipple or increase the size of the nipple opening Rationale: neonates who have VSD may fatigue quickly during feeding and ingest inadequate amounts. They should be monitored for weight gain and at least 6 wet diapers per day. A one month old should ingest 2-4 ounces of formula per feeding and progress to about 30 ounces per day by 4 months of age. Due to fatigue, the infant should rest, but feed at least every 2 hours to ensure adequate intake. A softer (preemie) nipple or a larger slit in the nipple helps to reduce the sucking effort and energy expenditure, thus allowing the infant to ingest more w/ less effort. Antibiotoic prophylaxis is recommended for infants w/ VSDs, but should not be mixed in a bottle of formula because it is difficult to ensure tha the total dose is consumed

Rheumatic fever

Most common cause of acquired heart disease Affects connective tissue, sore throat than fever Lots of rest, home school

A 3-month-old infant develops oral thrush. Which pharmacologic agent should the nurse plan to administer for treatment of this disorder?

Nystatin (Mycostatin).

Which preoperative nursing intervention should be included in the plan of care for an infant with pyloric stenosis?

Observe for projectile vomiting. Rationale: Projectile vomiting (D), the classic sign of pyloric stenosis, contributes to metabolic alkalosis.

An infant is receiving digoxin (Lanoxin) for congestive heart failure. The apical heart rate is assessed at 80 beats/min. What intervention should the nurse implement?

Obtain a therapeutic drug level. Rationale: Sinus bradycardia (heart rate < 90 to 110 beats/min in an infant) is an indication of digoxin toxicity, so assessment of the client's digoxin level has the highest priority.

Atrial septal defect

Opening in atria Acyanotic

The nurse is assessing the neurovascular status of a child in Russell's traction. Which finding should the nurse report to the healthcare provider?

Pale bluish coloration of the toes

When discussing discipline with the mother of a 4-year-old child, the nurse should include which guideline?

Parental control should be consistent.

A newborn female whose mother is HIV-positive is scheduled for the first follow-up assessment with the nurse. If the child is HIV-positive, which initial symptom is she most likely to exhibit?

Persistent cold Rationale: Respiratory tract infections commonly occur in the pediatric population, but the child with AIDS has a decreased ability to defend the body against these common infections. Thus, the most typical presenting symptom of a child who contracted AIDS through vertical transmission (i.e., from the mother during delivery) is a persistent cold or respiratory infection

A child with a permanent tracheostomy is confined to a wheelchair and is going to school for the first time tomorrow. During the school day, which intervention should be implemented for this child?

Place suctioning supplies on the back of the wheelchair when transporting. Rationale: Suctioning supplies should always be readily available for use with any client who has a tracheostomy.

A child breaks out with varicella infection (chickenpox) while hospitalized for a minor surgical procedure. Which intervention should the nurse implement first?

Place the child in strict isolation to prevent an outbreak on the unit. Rationale: The period of communicability of varicella is 2 days before the rash appears until all lesions are crusted; varicella is spread by direct or indirect contact of saliva or vesicles. Strict isolation is indicated to prevent further exposure to staff and others. Staff who have had varicella or the vaccine are not susceptible to contracting or spreading the virus and should be the only personnel assigned to care for this client

A 3-month-old infant returns from surgery with elbow restraints and a Logan's bow over a cleft lip suture line. Which intervention should the nurse implement to maintain suture line integrity during the initial postoperative period?

Place the infant upright in an infant seat position. Rationale: The use of an infant seat simulates a supine position with the head elevated and also prevents aspiration. Prone positioning should be avoided to prevent disruption of the protective Logan's bow and prevent the infant from rubbing the face on the bed surface. Mittens are not necessary and decrease the ability to provide sensory comfort, such as hand holding. Nasal suctioning should be avoided to prevent trauma or dislodging clots at the surgical site. Water-soluble lubricant will dry the suture line and cause crusting, which predisposes the suture line to poor healing and scarring.

A nurse provides the parents with information on health maintenance for their child with sickle cell disease. Which information reflected by the parents indicates understanding of the child's care?

Plenty of fluids should be consumed daily.

List the signs and symptosm of dehydration in an infant

Poor skin turgor, absence of tears, dry mucous membranes, weight loss, depressed fontanel, and decreased urinary output

While caring for a child on the oncology unit the nurse explains to the parents that total body irradiation is indicated for which for the following reasons? Palliative care Lymphoma therapy Definitive therapy for leukemia Preparation for bone marrow transplant

Preparation for bone marrow transplant

A 2-year-old child with trisomy 21 (Down syndrome) is brought to the clinic for a routine evaluation. Which assessment finding suggests the presence of a common complication often experienced by those with Down syndrome?

Presence of a systolic murmur Rationale: Congenital heart disease occurs in 40% to 50% of children with trisomy 21 (Down syndrome). Defects of the atrial or ventricular septum that create systolic murmurs are the most common heart defects associated with this congenital anomaly.

Following the reduction of an incarcerated inguinal hernia, a 4-month-old boy is scheduled for surgical repair of the inguinal hernia. Under which circumstance should the parents notify the health care provider prior to surgery?

Presence of an inguinal bulge after gentle palpation Rationale: The parents should notify the health care provider if the hernia remains irreducible after implementing simple measures, such as gentle palpation, warm bath, and comforting to reduce crying. If a loop of intestines is forced into the inguinal ring or scrotum and incarcerates, swelling can follow and possible strangulation of the bowel, intestinal obstruction, or gangrene of the bowel loop can occur, necessitating emergency surgical release.

An infant is born with a ventricular septal defect (VSD) and surgery is planned to correct the defect. The nurse recognizes that surgical correction is designed to achieve which outcome?

Prevent the return of oxygenated blood to the lungs.

When evaluating the effectiveness of interventions to improve the nutritional status of an infant with gastro-esophageal reflux, which intervention is most important for the nurse to implement?

Record weight daily.

The nurse is preparing a child with an intussusception for a prescribed barium enema. What is the main purpose of conducting this procedure prior to surgical intervention?

Reduce the invaginated bowel segment. Rationale: Intussusception, an invagination or telescoping of one portion of the intestine into another, causes intestinal obstruction in children (usually occurs between 3 months and 5 years of age). Nonsurgical treatment is attempted with hydrostatic pressure created by barium instillation, which often reduces the area of bowel intussusception, thereby negating the need for surgical intervention.

A 6-month-old male infant is admitted to the postanesthesia care unit with elbow restraints in place. He has an endotracheal tube and is ventilator-dependent but will be extubated soon following recovery from anesthesia. Which nursing intervention should be included in this child's plan of care?

Remove restraints one at a time and provide range-of-motion exercises. Rationale: Removing restraints one at a time is safer than simultaneously. The infant should have the restrained extremities assessed frequently for signs of neurologic or vascular impairment, and range-of-motion exercises should be performed with these assessments. Under no circumstances should restraints be applied to the client continuously. Documentation of assessment findings regarding the restrained extremities must occur much more frequently than every 72 hours; however, the reason for using restraints must be justified and should be stated in the medical record.

The nurse is teaching an adolescent girl with scoliosis about a Milwaukee brace that her health care provider has prescribed. Which instruction should the nurse provide to this client?

Remove the brace 1 hour each day for bathing only. Rationale: The Milwaukee brace is designed to slow the progression in spinal curvature while the adolescent is growing. The brace should be worn 23 hours a day and removed a total of 1 hour a day for hygiene. There are no specific exercises for increasing the range of motion in the back that should be performed. A T shirt should be worn next to the body and the brace put on over the T shirt to protect the skin. The brace will not cure the spinal curvature but should slow the progression of the scoliosis.

To take the vital signs of a 4-month-old child, which order provides the most accurate results?

Respiratory rate, heart rate, then rectal temperature

An 18-month-old is admitted to the hospital with possible Hirschsprung's disease. When obtaining a nursing history, the nurse asks about bowel habits. What description of the disease?

Ribbon-like and brown.

decerebrate posturing

Rigid extension and pronation of the arms and legs

The nurse reviews the latest laboratory results for a child who received chemotherapy last week and identifies a reduced neutrophil count. Which nursing diagnosis has the highest priority for this child?

Risk for infection

Which is a priority problem for a child with severe edema caused from nephrotic syndrome?

Risk for skin breakdown

what disease occurs with vitamin C deficiency?

Scurvy

The vital signs of 4 yr old child w/ polyuria are: BP 80/40, pulse, 118, and Resp. 24. The child's pedal pulses are present w/ a volume of +1, and no edema is observed. What action should the nurse implement first?

Start an IV infusion of normal saline

A child comes to the school nurse complaining of itching. Further assessment reveals that the child has impetigo. What action should the nurse take?

Send the child home with the parents to see the health care provider before returning to school. Rationale: Impetigo is a staphylococcal infection and is transmitted by person-to-person contact. The child should be sent home with a note to the parents explaining the condition

A 5-month-old is admitted to the hospital with vomiting and diarrhea. The pediatrician prescribes dextrose 5% and 0.25% normal saline with 2 mEq KCl/100 ml to be infused at 25 ml/hour. Prior to initiating the infusion, the nurse should obtain which assessment finding?

Serum BUN and creatinine levels

The nurse caring for a 4yo female in the ER is about to start an IV. The nurse's best method for explaining the procedure to the child is to:

Show the child the IV placement equipment, and demonstrate the procedure on the doll.

An 18-month-old child returns to the unit following a cardiac catheterization with a cannulated femoral artery site. Which intervention should the nurse implement?

Show the parents how to hold the child with the extremity extended. Rationale: The extremity should be extended to prevent trauma to the femoral catheterization site

The nurse notes that a 16-year-old male client is refusing visits from his classmates. Further assessment reveals that he is concerned about his edematous facial features. Based on these assessment findings, the nurse should plan interventions related to which nursing diagnosis?

Social isolation Rationale: Peer acceptance and body image are significant issues in the growth and development of adolescents. The answer addresses the problem of a lack of contact with peers stemming from his desire to protect his ego.

Which assessment finding(s) should the nurse expect when caring for a child with cystic fibrosis? (Select all that apply.)

Steatorrhea Foul-smelling stools Delayed growth Pulmonary congestion

A child with cystic fibrosis is having stools that float and are foul smelling. Which descriptive term should the nurse use to document the finding?

Steatorrhea.

During administration of a blood transfusion, a child complains of chills, headache, and nausea. Which action should the nurse implement?

Stop the infusion immediately and notify the healthcare provider

During administration of a blood transfusion, a child complains of chills, headache, and nausea. Which action should the nurse implement?

Stop the infusion immediately and notify the healthcare provider.

The nurse is preparing a health teaching program for parents of toddlers and preschoolers and plans to include information about the prevention of accidental poisonings. It is most important for the nurse to include which instruction?

Store all toxic agents and medicines in locked cabinets. Rationale: The only reliable way to prevent poisonings in young children is to make the items inaccessible

A father of a 5-year-old boy calls the nurse to report that his son, who has had an upper respiratory infection, is complaining of a headache, and his temperature has increased to 103° F, taken rectally. Which intervention has the highest priority?

Tell the parent to take the child to the emergency department. Rationale: The child is exhibiting symptoms that may indicate possible meningitis, and the parents should be encouraged to get immediate evaluation

Which class of antiinfective drugs is contraindicated for use in children under 8 years of age?

Tetracyclines.

A community health nurse is providing instructions to a group of mothers regarding the safe use of car seats for toddlers. The nurse determines that the mother of a toddler understands the instructions if the mother states which of the following? a. The care seat can be placed in a face-forward position when the height of the toddler is 27 inches b. The car seat should never be placed in a face-forward position c. the car seat can be placed in a face-forward position at any time d. The car seat is suitable for the toddler until the toddler reaches the weight of 40 pounds

The car seat is suitable for the toddler until the toddler reaches the weight of 40 pounds

Why are IV fluids important for a child with an increased respiratory rate?

The child is at risk for dehydration and acid-base imbalances

Kernig's sign

The child is not able to extend the leg when the thigh is flexed anteriorly at the hip.

An ER nurse is assessing a 12-month old female. Which statement accurately describes the best method for assessing this child?

The nurse should assess the child while she is in her mother's lap.

How can the nurse best facilitate the trust relationship between infant and parent while the infant is hospitalized?

The nurse should encourage the parents to hold their child as much as possible.

An 11yo male is being evaluated in the ER for an inguinal hernia. Which statement accurately describes how the nurse should approach him for his physical assessment?

The nurse should explain to the child what the nurse will be doing in basic understandable terms.

A premature newborn girl, born 24 hours ago, is diagnosed with a patent ductus arteriosus (PDA) and placed under an oxygen hood at 35%. The parents visit the nursery and ask to hold her. Which response should the nurse provide to the parents?

The oxygen hood is holding the baby's oxygen level just at the point which is needed. You may stroke and talk to her.

an infant is placed in a pavlik harness for developmental dysplasia of the hip . which of the following statements made by a parent indicates correct knowledge of the care of this infant? The straps of the harness should be placed next to the skin The harness should be worn for 6 hours a day It will take a long time for my child to walk and crawl I can move my child around on a large skateboard.

The straps of the harness should be placed next to the skin***

The clinic nurse is taking the hx for a new 6-month-old client. The mother reports that she took a great deal of aspirin while pregnant. Which assessment should the nurse obtain?

Type of reaction to loud noises

Prophylactic antibiotics are prescribed for a child who has mitral valve damage. The nurse should advise the parents to give the antibiotics prior to which occurrence?

Urinary catheterization Rationale: Prophylactic antibiotics are usually prescribed prior to any invasive procedure for children who have valvular damage.

The nurse assigns an unlicensed assistive personnel (UAP) to provide morning care to a newly admitted child with bacterial meningitis. What is the most important instruction for the nurse to review with the UAP?

Use designated isolation precautions.

A female teenager is taking oral tetracycline HCL (Achromycin V) for acne vulgaris. What is the most important instruction for the nurse to include in this client's teaching plan?

Use sunscreen when lying by the pool.

list the four defects associated with tetralogy of fallot

VSD overriding aorta pulmonary stenosis right ventricular hypertrophy

The mother of a 2-year-old boy consults the nurse about her son's increased temper tantrums. The mother states, "yesterday he threw a fit in the grocery store, and I did not know what to do. I was so embarrassed. What can I do if this occurs again?" Which recommendation is best for the nurse to provide this mother?

Walk away from him and ignore the behavior

A 14-year-old female client tells the nurse that she is concerned about the acne she has recently developed. Which recommendation should the nurse provide?

Wash the hair and skin frequently with soap and hot water.

A child falls on the playground and is brought to the school nurse with a small laceration on the forearm. Which action should the nurse implement first?

Wash the wound gently with mild soap and water.

"A 4-year-old child with cancer is admitted to the hospital for radiation therapy and surgery. To assess adequacy of support for the child's psychosocial needs, the nurse would ask the parents which question?" a. what signs and symptoms has your child been having? b. Will a family member be able to stay with the child most of the time? c. How long have you known your child's diagnosis? d. what are your child's favorite books, activities and toys?

Will a family member be able to stay with the child most of the time?

Which measures should be used to accurately calculate a pediatric medication dosage?

a child's height & weight Body surface area of child nomogram determined mathematical constant

describe the pathophysiology of vesicoureteral reflux

a malfunction of the valves at the end of the ureters, allowing urine to reflux out of the bladder into the ureters and possibly into the kidneys

Differentiate between right-to-left and a left-to-right shunt in cardiac disease

a right-to-left shunt bypasses the lungs and delivers unoxygenated blood to the systemic circulation causing cyanosis a left-to-right shunt moves oxygenated blood back through the pulmonary circulation

A nurse is reviewing the results of a sweat test performed on a child with cystic fibrosis (CF). The nurse would expect to note which finding? a. a sweat sodium concentration less than 40 mEq/L b. a sweat potassium concentration less than 40 mEq/L c. a sweat potassium concentration greater than 40 mEq/L d. a sweat sodium concentration greater than 60 mEq/L

a sweat sodium concentration greater than 60 mEq/L

The nurse is assigning care for a 4-year-old child with otitis media and is concerned about the child's increasing temperature over the past 24 hours. When planning care for this child, it is important for the nurse to consider that

a tympanic measurement of temperature will provide the most accurate reading

The nurse is assessing an 8 month old child who has a medical diagnosis of tetrology of Fallot. Which symptom is the client most likely to exhibit?

clubbed fingers

Describe the sequence of events in a vaso-occulsive crisis in sickle cell anemia

a vaso-occulsive crisis is caused by the clumping of red blood cells, which blocks small blood vessels; therefore, the cells cannot get through the capillaries, causing pain and tissue and organ ischemia. Lowered oxygen tension affects HgbS, which causes sickling of the cells

Which of the following assessment findings would the nurse expect to find in the school-age child with Duchenne's muscular dystrophy? a. Enlargement of muscles b. Bedridden c. Weak cough reflex d. Paralysis of lower muscles

a. Enlargement of muscles** Enlarged with fatty deposits

The nurse received a lab report stating a child w/ asthma has theophyline level of 15 mcg/dl. What action will the nurse take?

a. Hold the next dose of theophylline Therapeutic levels of theophylline is 10-20 mcg/dl, so the child's level is w/in the therapeutic rage.

Which menu selection by a child w/ celiac disease indicates to the nurse that the child understands necessary dietary considerations?

a. Oven baked potato chips & cola Celiac disease causes an intolerance to the protein gluten found in oats, rye, wheat, and barley. The child should avoid any produces containing these indredients to avoid symptoms such as diarrhea.

A nurse prepares to administer digoxin (Lanoxin) to a newborn infant with a diagnosis of congestive heart failure. The nurse notes that the apical rate is 140 beats per minute. Which of the following nursing actions is appropriate? a. administer the digoxin because the apical rate is within normal limits b. recheck the apical rate in 1 hour and administer the medication at that time c. notify the physician because the apical rate is lower than the normal range d. hold the medication, because the apical rate is normal, indicating that the medication is not needed

a. administer the digoxin because the apical rate is within normal limits

"A hospitalized toddler cries when anyone enters the room, and kicks, yells, and clings to the parents if they try to leave. Based on these data, the nurse determines that the priority nursing diagnosis is:" a. fear related to unfamiliar surroundings b. deficient divisional activity related to developmental stage c. compromised family coping related to the sick child d. delayed growth and development related to overprotective parenting

a. fear related to unfamiliar surroundings

A 4-year-old child is reluctant to take deep breaths after abdominal surgery. The most effective measure to encourage deep breathing is to: a. have the child pretend he is the big, bad wolf blowing the little pig's house down b. give the child colorful latex balloons to blow up c. tell the child to exhale forcefully through the peak flow meter d. administer chest percussion in several postural drainage positions

a. have the child pretend he is the big, bad wolf blowing the little pig's house down

A 9-month-old infant is admitted to a pediatric unit with a diagnosis of dehydration and malnutrition and suspected failure to thrive. Child neglect is suspected. Which of the following would be most important for the nurse to observe when the parents visit the infant? a. the parents' level of concern about the child b. the parents' patterns of visitation c. the parents' interactions with one another d. clues regarding the nutritional patterns of the other children in the family

a. the parents' level of concern about the child

"A client with gastroesophageal reflux disease (GERD) complains of chest discomfort that feels like heartburn, especially following each meal. After teaching the client to take antacids as prescribed, the nurse suggests that the client lie in which position during sleep?" a. with the head of the bed elevated 6 to 8 inches b. flat c. supine with the head of the bed flat d. on the stomach with the head of the bed flat

a. with the head of the bed elevated 6 to 8 inches

What are the signs and symptoms of compartment syndrome?

abnormal neurovascular assessment; cold extremity, severe pain, inability to move the extremity and poor capillary refill

A nurse is caring for a child after an inguinal hernia repair. Which finding would indicate that the surgical repair was effective? a. abdominal distention b. absence of inguinal swelling with crying c. a clean, dry incision d. an adequate flow of urine

absence of inguinal swelling with crying

A nurse prepares to administer digoxin (Lanoxin) to a newborn infant with a diagnosis of congestive heart failure. The nurse notes that the apical rate is 140 beats per minute. Which of the following nursing actions is appropriate? a. administer the digoxin because the apical rate is within normal limits b. recheck the apical rate in 1 hour and administer the medication at that time c. notify the physician because the apical rate is lower than the normal range d. hold the medication, because the apical rate is normal, indication that the medication is not needed

administer the digoxin because the apical rate is within normal limits

A 6 month old boy and his mother are at healthcare provider's office for a well-baby check up and routine immunizations. The healthcare provider recommends to the mother that the child receive an influenza vaccine. What medications should the nurse plan to administer today?

all the immunizations w/ the influenza vaccine given at a separate site fr. any other injection At 6 months of age, the routine immunizations should HEP B, DTaP, Hib, PCV (pneumococcal) , IPV (inactivated poliovirus) and influenza. The influenza vaccine should be given at a separate site fr. any other injection.

"When administering a liquid medication to an uncooperative toddler, the nurse would implement which strategy?" a. allow the parents to remain the room b. remove the child to another room away from the parents c. restrain the child in a high chair d. restrain the child in a papoose restraining device

allow the parents to remain the room

What antibiotics are usually prescribed for bacterial meningitis?

ampicilin, ceftriazzone, chloramphenicol

Nursing interventions and medical treatment for a child with leukemia are based on what three physiologic problems?

anemia (decreased erythrocytes) infection (neutropenia) bleeding thrmbocytopenia (decreased platelets)

What early signs should the nurse assess for if lead poisoning is suspected?

anemia, acute cramping, abdominal pain, vomiting, constipation, anorexia, headache, lethargy, hyperactivity, aggression, impulsiveness, decreased interest in play, irritability, short attention span

list the signs and symptoms of iron deficiency

anemia; pale conjunctive; pale skin; atrophy of papillae on tongue; brittle, ridged, or spoon-shaped nails; and thyroid edema

what cardiac complications are associated with rheumatic fever?

aortic valve stenosis and mitral valve stenosis

The nurse is assessing a 13 yr old girl w/ susptected hyperthyroidism. Which question is most important for the nurse to ask her during the admission interview?

are you experiencing any type of nervousness?

list five risks in cardiac catheterizaiton

arrhythmia bleeding perforation phlebitis obstruction of the arterial entry site

A nurse is preparing to care for a child from a culture different from the nurse's. What is the best way to address the cultural needs of the child and family when the child is admitted to the health care facility? a. ask questions and explain to the family why the questions are being asked b. explain to the family that while the child is being treated, they need to discontinue cultural practices because they may be harmful to the child c. ignore cultural needs because they are not important to health care professionals d. only address those issues that directly affect the nurse's care of the child

ask questions and explain to the family why the questions are being asked

All of the following interventions can be used to evaluate the effectiveness of nursing and medical interventions used to treat diarrhea. Which intervention is least useful in the nurse's evaluation of a 20 month old child?

assessing fontanels

What interventions can be taught to prevent UTI in children?

avoid bubble baths; void frequently; drink adequate fluids, especially acidic fluids such as apple or cranberry juice; and clean genital area form front to back

A nurse instructs a mother on measures to take to reduce the incidence of gastroesophageal reflux (GER) in a child. Which statement by the mother indicates a need for further teaching? a. I will give my child small feedings often throughout the day b. I will buy bottle nipples that have smaller holes for my child c. I will add a small amount of cereal to my child's formula d. I will give my child a pacifier and maintain an upright position after meals

b. I will buy bottle nipples that have smaller holes for my child

An infant with AIDS will be attending daycare the daycare workers are concerned about spreading the virus. The public health nurse explains the precautions the workers should take. These precautions include: a. Storing all of the infant's supplies separately from the other childrens' b. Wearing gloves when changing the child's diaper c. Always wearing gloves and isolation gowns when handling the infant d. Minimizing contact with the infant when it is febrile—baby is immunocompromised

b. Wearing gloves when changing the child's diaper

"A 4-year-old child with cancer is admitted to the hospital for radiation therapy and surgery. To assess adequacy of support for the child's psychosocial needs, the nurse would ask the parents which question?" a. what signs and symptoms has your child been having? b. Will a family member be able to stay with the child most of the time? c. how long have you known your child's diagnosis? d. What are your child's favorite books, activities and toys?

b. Will a family member be able to stay with the child most of the time?

A nurse develops a plan of care for a one-month-old infant hospitalized for intussusception. Which nursing measure would be most effective to provide psychosocial support for the parent-child relationship? a. encourage the parents to go home and get some sleep b. encourage the parents to room-in with their infant c. provide educational materials d. initiate home nutritional support as early as possible

b. encourage the parents to room-in with their infant

A client with gastroesophageal reflux disease (GERD) has just received a breakfast tray. The nurse setting up the tray for the client notices that which of the following foods is the only one that will increase the lower esophageal sphincter (LES) pressure and thus lessen the client's symptoms? a. fresh scrambled eggs b. nonfat milk c. whole wheat toast with butter d. coffee

b. nonfat milk

A 4-year-old child who was recently hospitalized is brought to the clinic by his mother for a follow-up visit. The mother tells the nurse that the child has begun to wet the bed ever since the child was brought home from the hospital. The mother is concerned and asks the nurse what to do. The appropriate nursing response is which of the following? a. you need to discipline the child b. this is a normal occurrence following hospitalization c. we will need to discuss this behavior with the physician d. the child probably has developed a urinary tract infection

b. this is a normal occurrence following hospitalization

when discussing scoliosis screening with the parent of a 12 year old child the school nurse explains that symptoms typical of adolescent idiopathic structural scoliosis includes all of the following except back pain skirts that hand unevenly unequal shoulder heights uneven waist angles

back pain***don't typically have back pain

Why is genetic counseling important for the family of a child with cystic fibrosis?

because the disease is autosomal recessive in its genetic pattern

Methylphenidate (Ritalin) is prescribed for a child with a diagnosis of attention deficit hyperactivity disorder (ADHD). The nurse provides instructions to the mother regarding the administration of the medication and tells the mother to administer the medication: a. before dinner and at bedtime b. before breakfast and before the noontime meal c. in the morning after breakfast and at bedtime d. at the noontime and evening meals

before breakfast and before the noontime meal

How should burns in child be assessed?

by using the Lund-Browder chart, which takes into account the changing proportions of the child's body

"A nurse gathers assessment data from a client admitted to the hospital with gastrointestinal reflux disease (GERD) who is scheduled for a Nissen fundoplication. Based on an understanding of this disease, the nurse determines that the client may be at risk for which complication?" a. diarrhea b. belching c. aspiration d. abdominal pain

c. aspiration

A neonate is born with mild clubfeet. When the parents ask the nurse how this will be corrected, the nurse should explain that a. traction is tried first b. surgical intervention is needed c. frequent, serial casting is tried first d. children outgrow this condition when they learn to walk

c. frequent, serial casting is tried first

When giving an intramuscular injection to a 4-year-old child the nurse should: a. use the vastus lateralis muscle only b. allow the child to choose between a lying or a standing position c. obtain assistance to administer the injection d. distract the child with conversation or a toy

c. obtain assistance to administer the injection

A nurse is preparing a plan of care for a child being admitted to the hospital with a diagnosis of congestive heart failure (CHF). The nurse avoids including which of the following in the plan? a. limiting the time the child is allowed to bottle-feed b. elevating the head of the bed c. walking the child for feeding to ensure adequate nutrition d. providing oxygen during stressful periods

c. walking the child for feeding to ensure adequate nutrition

A mother of a 3-year-old child calls a neighbor who is a nurse and tells the nurse that her child just ate the mouse poison that was stored in a cabinet. The nurse would instruct the mother to take what action immediately? a. call the poison control center b. give the child a glass of carbonated beverage to drink c. take the child to the ER d. try to determine how much mouse poison the child consumed

call the poison control center

A nurse is providing home care instructions to the parents of an infant who had surgical repair of an inguinal hernia. The nurse instructs the parents to do which of the following to prevent infection at the surgical site? a. change the diapers as soon as they become damp b. report a fever immediately c. soak the infant in a tub bath twice a day for the next 5 days d. restrict the infant's physical activity

change the diapers as soon as they become damp

The nurse is caring for a 12 year-old w/ Syndrome of Inappropriate Antidiuretic Hormone (SIADH). This child should be carefully assessed for which complication?

changes in LOC

What discharge instructions should be included concerning a child with spica cast?

check child's circulation keep cast dry do no place anything under cast prevent cast soilage during toileting or diapering do not turn child using an abductor bar

Describe the preoperative nursing care for a child with Hirschsprung disease

check vital signs and take axillary temperatures provide bowel cleansing program and teach about colostomy observe for bowel perforation measure abdominal girth

list the signs and symptoms of esophageal atresia with TEF

chocking, coughing, cyanosis, and excess salivaiton

A 2-year-old child with gastro-esophageal reflux has developed a fear of eating. What instruction should the nurse include in the parents' teaching plan?

consistently follow a set mealtime routine.

A nurse instructs a parent regarding the appropriate actions to take when the toddler has a temper tantrum. Which statement by the parent indicates a successful outcome of the teaching? a. I will send my child to a room alone for 10 minutes after every tantrum b. I will reward my child with candy at the end of each day without a tantrum c. I will give frequent reminders that only bad children have tantrums d. I will ignore the tantrums as long as there is no physical danger

d. I will ignore the tantrums as long as there is no physical danger

A nurse caring for a child with congestive heart failure provides instructions to the parents regarding the administration of digoxin (Lanoxin). Which statement by the mother indicates a need for further instructions? a. if my child vomits after I give the medication, I will not repeat the dose b. I will check my child's pulse before giving the medication c. I will check the dose of the medication with my husband before I give the medication d. I will mix the medication with food

d. I will mix the medication with food

The mother of a newborn is upset that her baby was diagnosed with congenital clubfoot. She asks the nurse what she did to cause her baby's deformity. Which of the following responses is the most appropriate? a. Abnormal uterine positioning could have caused this deformity b. A lack of good nutrition during pregnancy could have caused this defect c. Having the baby before the due date could have caused this problem d. There are no known etiologies of the defect.

d. There are no known etiologies of the defect.

A nurse is caring for a child with a diagnosis of congestive heart failure (CHF). The nurse avoids which action in caring for the child? a. allowing uninterrupted rest periods b. limiting the time the child is allowed to bottle-feed c. providing oxygen during stressful periods d. keeping the head of the bed flat

d. keeping the head of the bed flat

The parents of a 3 week old infant report that the child eats well but vomits after each feeding. What information is most important for the nurse to obtain?

description of vomiting episodes in past 24 hrs

what are common signs of digoxin toxicity?

diarrhea fatigue weakness nausea vomiting check for bradycardia before administering

describe the mechanism of inheritance of Duchenne muscular dystrophy

duchenne muscular dystrophy is inherited as an X-linked recessive trait

A nurse develops a plan of care for a one-month-old infant hospitalized for intussusception. Which nursing measure would be most effective to provide psychosocial support for the parent-child relationship? a. encourage the parents to go home and get some sleep b. encourage the parents to room-in with their infant c. provide educational materials d. initiate home nutritional support as early as possible

encourage the parents to room-in with their infant

a 6- year old admitted to the pediatric unit after falling of a bicycle. Which intervention should the nurse implement to assist the child's adjustments to hospitalization?

establishing an individual schedule familiarizes the child to the hospital environment and decreases anxiety.

What care is needed for a client with a temporary colostomy?

family needs education about skin care and appliances referral to an enterostomal therapist is appropriate

Describe the signs and symptoms of a child with meningitis

fever, irritability, vomiting, neck stiffness, opisthotonos, positive Kernig sign, positive brudzinski signs; infant may not show all

A toddler is admitted to the hospital for fever of unknown origin. The mother's time at the hospital is limited to the hours that her other children are in school. The nurse shows an understanding of a toddler's psychosocial development by making which statement to the mother? a. it is better to leave without saying good-bye, so your child will not be upset b. your child is too old to be having separation anxiety. Crying is just a way for children to control parents c. your child is egocentric, which allows a child to self comfort d. games like peek-a-boo and hide and seek will help your child understand that you will return

games like peek-a-boo and hide and seek will help your child understand that you will return

what are the side effects of Dilantin?

gingival hyperplasia dermatitis ataxia GI distress

describe the information families should be given when a child is receiving oral iron prepartions

give oral iron on an empty stomach and with vitamin C use straws to avoid discoloring teeth tarry stools are normal increase dietary sources of iron

Describe nursing interventions to reduce the workload of the heart

give small, frequent feedings or gavage feedings plan frequent rest periods maintain a neutral thermal environment organize activities to disturb child only as indicated

The nurse is assessing a 2 year old. What behavior indicates that the child's language development is within normal limits?

half of a child's speech is understandable Between approximately 15 & 24 months of age, a child's speech is only ½ understandable

A nurse is assessing a child admitted to the hospital with a diagnosis of rheumatic fever. The nurse asks the child's mother which significant question during the assessment? a. has your child had difficulty urinating? b. has any family member had a sore throat within the past few weeks? c. has any family member had a gastrointestinal disorder in the past few weeks? d. has your child been exposed to anyone with chickenpox?

has any family member had a sore throat within the past few weeks?

A nurse is collecting data on a child suspected of having rheumatic fever. The nurse plans to obtain specific data regarding recent illnesses in the child and asks the parent which question? a. has the child had a recent streptococcal infection of the throat? b. has the child had a recent ear infection? c. has the child had a recent case of otitis media? d. has the child had a recent case of pneumonia

has the child had a recent streptococcal infection of the throat?

Children with chronic otitis media are at risk for developing what problem?

hearing loss

A home care nurse is instructing the mother of a child with cystic fibrosis (CF) about the appropriate dietary measures. The nurse tells the mother that the child needs to consume a: a. low calorie, low fat diet b. high calorie, high protein diet c. low calorie, low protein diet d. high calorie, restricted fat

high calorie, high protein diet

explain why hydration is a priority in treating sickle cell disease

hydration promotes hemodilution and circulation of the red cells through the blood vessels

Differentiate the signs of hypoglycemia and hyperglycemia

hypoglycemia: tremors, sweating, headache, hunger, nausea, lethargy, confusion, slurred speech, anxiety, tingling around mouth, nightmares hyperglycemia: polydipsia,polyuria, ployphagia, blurred vision, weakness, weight loss, and syncope

what is the physiologic reason for the lab finding of hypoproteinemia in nephrosis?

hypoproteinemia occurs because the glomeruli are permeable to serum protieins

list two contraindications to live virus immunization

immunocompromised child a child in a household with an immunocompromised individual

A child is admitted to the hospital with a diagnosis of acute rheumatic fever. The nurse analyzes the laboratory results and determines that which of the following findings would confirm the likelihood of acute rheumatic fever? a. increased leukocyte count b. decreased hemoglobin count c. increased antibody level d. decreased erythrocyte sedimentation rate

increased antibody level

A child is admitted to the hospital with a diagnosis of acute rheumatic fever. The nurse reviews the blood laboratory findings knowing that which of the following will confirm the likelihood of this disorder? a. increased leukocyte count b. decreased hemoglobin count c. increased antistreptolysin-O (ASO) d. decreased erthrocyte sedimentation rate

increased antistreptolysin-O (ASO)

A 2-year-old child is admitted to the hospital with juvenile rheumatoid arthritis (JRA). During the focused assessment, the nurse makes it a priority to note the presence of which of the following?" a. increased irritability and the child's insistence to be carried out b. complaints of joint stiffness c. history of daily temperature elevations d. description of how difficult it is to move around after periods of inactivity

increased irritability and the child's insistence to be carried out

What teaching should parents of a newly shunted child receive?

information about signs of infection and increased ICP; understanding that shunt should not be pumped and that child will need revisions with growth; guidance concerning growth and development

List the signs and symptoms of increased ICP in older children

irritability change in LOC motor dysfunction headache vomiting unequal pupil response seizures

"A newborn infant is diagnosed with hypospadias, and the mother asks the nurse about the disorder. The nurse bases the response on which of the following?" a. it is a congenital anomaly in which the actual opening of the urethra meatus is below the normal placement on the glans penis b. it occurs when one or both testes fail to descend through the inguinal canal into the scrotal sac c. it is a congenital anomaly in which the actual opening of the urethral meatus is dorsal to the urethral opening d. it is a congential anomaly characterized by the extrusion of the urinary bladder to the outside of the body

it is a congenital anomaly in which the actual opening of the urethra meatus is below the normal placement on the glans penis

what is the genetic transmission pattern of hemophilia?

it is an X-linked recessive chromosomal disorder transmitted by the mother and expressed in male children

A student nurse examines an Asian American infant's eyes and notes that the infant's eyes are crossed. The registered nurse asks the student to interpret the finding. Which statement by the student indicates an understanding of this assessment finding? a. it probably isn't strabismus but appears that way because of the child;s ethnic background b. you will want to call the pediatrician immediately because this could lead to detached retina c. it probably is strabismus because the baby's mother has abused tranquilizers d. strabismus isn't life threatening but it requires surgery in the first 2 months to prevent the crossed eyes from being a life long condition

it probably isn't strabismus but appears that way because of the child;s ethnic background

What are the symptoms of congenital hypothyroidism in early infancy?

large, protruding tongue; coarse hair; lethargy; sleepiness and constipation

what should families and clients do to avoid triggering sickling episodes?

keep child well hydrated avoid known sources of infection avoid high altitudes avoid strenuous exercises

A nurse is caring for a child with a diagnosis of congestive heart failure (CHF). The nurse avoids which action in caring for the child? a. allowing uninterrupted rest periods b. limiting the time the child is allowed to bottle-feed c. providing oxygen during stressful periods d. keeping the head of the bed flat

keeping the head of the bed flat

"A client is unable to expectorate sputum for a sputum sample, and the nurse is preparing to obtain the sample via saline inhalation. The nurse instructs the client to inhale the warm saline vapor via nebulizer by:" a. hold the nebulizer under the nose b. keeping the lips closed lightly over the mouthpiece c. keeping the lips closed tightly over the mouthpiece d. alternating one vapor breath with one breath from room air

keeping the lips closed lightly over the mouthpiece

what position would best relieve the child experiencing a tet spell?

knee-chest position or squatting

how should a parent be instructed to child-proof a house?

lock all cabinets safely store all toxic household items in a locked cabinets examine the house from the child's point of view

Describe safe monitoring of prednisone administration and withdrawal

long-term prednisone should be given every other day. Signs of edema, mood changes, and GI distress should be noted and reported. The drug should be tapered, no discontinued suddenly

list the laboratory findings that can be expected in a dehydrated child

loss of bicarbonated/decreased pH loss of sodium (hyponatremia) loss of potassium (hypokalemia) elevated Hct and elevated BUN

A mother of a toddler who is hospitalized with mild dehydration must leave her child to go to work. Which behavior would the nurse expect to observe in the toddler immediately after the mother's departure? a. silently curled in bed with a blanket b. loudly crying and kicking both legs c. playing quietly with a favorite toy d. sucking thumb and rocking back and forth

loudly crying and kicking both legs

Describe the postoperative nursing care for an infant with pyloric stenosis

maintain IV hydration and provide small, frequent feedings of glucose or electrolyte solutions or both within 4 to 6 hours. Gradually increase to full-strength formula Position infant on right side in semi-fowler position after feeding

What nursing actions are initiated for the newborn with suspected esophageal atresia with TEF?

maintain NPO immediately and suction secretions

what are the priorities for a child undergoing abdominal surgery?

maintain fluid balance (I/O, NG suction, monitor electrolytes); monitor vital signs; care for drains; if present assess bowel function; prevent infection of incision area and other postoperative complications; and support child and family with appropriate teaching

State the three main gals in providing nursing care for a child experiencing a seizure

maintain patent airway, protect form injury and observe carefully

list dietary sources of iron

meat, green leafy vegetables, fish, liver, whole grains, legumes

List foods high in phenylalanine content

meat, milk, diary products, and eggs

Which finding in a 19 yr old female client should trigger further assessment by the nurse?

menstruation has not occurred Menstruation is an expected secondary sex characteristic that occurs with pubescence and typically occurs by age 18, so A should prompt further investigation to determine the cause of this primary amenorrhea. Children receive tetanus as part of the DPT childhood immunization series, and a booster is not typically given until age 16.

describe the care of a child in a mist tent

monitor child's temperature, keep tent edges tucked in, keep clothing dry, assess respiratory status, look at child inside tent

How can the nurse best evaluate the adequacy of fluid replacement in children?

monitoring urine output

describe developmental factors that would impact the school-age child with diabetes

need to be like peers; assuming responsibility for own care; modification of diet; snacks and exercise in school

How is congenital hypothyroidism diagnosed?

newborn screening revealing a low T4 and a high TSH

What are signs of anorectal malformation?

newborn who does not pass meconium within 24 hours; meconium appearing through a fistula or in the urine; an unusual appearing and dimple

A nurse notes that a client's eyes are continuously moving back and forth within the eye sockets. The nurse documents in the medical record that the client has: a. ataxia b. nystagmus c. pronator drift d. hyperreflexia

nystagmus

during an admission assessment on an 8 year old child who has acquired immunodeficiency syndrome (AIDS) the nurse records the following vital signs: bp 116/78, aprical pulse 99, respirations 24, and an oral temperature of 100.2F. which finding should be reported to the physician immediately ?

oral temperature of 100.2F

A clinic nurse is performing an assessment on a child. Which finding indicates the presence of an inguinal hernia? a. painless inguinal swelling that appears when the child cries or strains b. complaints of difficulty defecating c. complaints of a dribbling urinary stream d. absence of the tests with scrotum

painless inguinal swelling that appears when the child cries or strains

What nutritional support should be provided for a child with cystic fibrosis?

pancreatic enzyme replacement, fat-soluble vitamins, and moderate to low carbohydrates, high protein, moderate to high fat diet

A nurse is assessing a child after hydrostatic reduction for intussusception. The nurse would expect to observe which finding after this procedure? a. severe colicy-type pain with vomiting b. currant jelly like stools c. passage of barium or water soluble contrast with stools d. severe abdominal distention

passage of barium or water soluble contrast with stools

"When obtaining a history from parents of a 5 month old child suspected of having intussusception, which assessment area would be most important for the nurse to address?" a. pattern of abdominal pain b. known allergies c. dietary intake during the past 24 hours d. usual pattern of bowel movements

pattern of abdominal pain

A clinic nurse is providing home care instructions to the mother of a 3-year-old child with a diagnosis of vomiting and diarrhea due to gastroenteritis. The nurse instructs the mother to give the child which of the following to maintain hydration status? a. popsicles b. soda pop c. apple juice d. pedialyte

pedialyte (oral electrolyte solution)

What medications are used to treat rheumatic fever?

penicllin, erythromycin, and aspirin

Breathing exercises and postural drainage is prescribed for a child with cystic fibrosis. A nurse implements these procedures by telling the child to: a. perform the postural drainage, then the breathing exercises b. perform the breathing exercises, then the postural drainage c. schedule the procedures so they are 4 hours apart d. perform postural drainage in the morning and breathing exercises in the evening

perform the postural drainage, then the breathing exercises

A 5-year-old child is admitted to the hospital for heart surgery to repair the tetralogy of Fallot. The nurse reviews the child's record and notes that the child has clubbed fingers. The nurse understands that the clubbing is most likely caused by: a. peripheral hypoxia b. delayed physical growth c. chronic hypertension d. destruction of bone marrow

peripheral hypoxia

list three classic signs and symptoms of measles

photophobia confluent rash that begins on the face and spreads downward Koplik spots on the buccal mucosa

The nurse is planning the care of a 2 year old w/ severe eczema on the face, next, and scalp fr. scratching the affected areas. Which nursing intervention is most effective in preventing further excoriation due to the purities?

place elbow restraints on the child's arms.

A home care nurse visits a 3-year-old child with chickenpox. The child's mother tells the nurse that the child keeps scratching the skin at night and asks the nurse what to do. The nurse tells the mother to: a. apply generous amounts of cortisone cream to prevent itching b. place soft cotton gloves on the child's hands at night c. keep the child in a warm room at night so the covers will not cause the child to scratch d. give the child a glass of warm milk at bedtime to help the child sleep

place soft cotton gloves on the child's hands at night

The nurse is planning care for school-aged children @ a community care center. Which activity is best fo the children?

playing follow the leader

The nurse is teaching a 12 yr old male adolescent and his family about taking injections of growth hormone for idiopathic hypopituitarism. Which adverse symptoms, commonly associated w/ growth hormone therapy, should the nurse plan to describe to the child and his family?

polyuria/polydipsia

list the common signs of cardiac problems in an infant

poor feeding poor weight gain respiratory distress infections edema cyanosis

Explain why hypospadias correction is performed before the child reaches preschool age

preschoolers fear castration, achieving sexual identity, and acquiring independent toileting skills

What care is indicated for a child with juvenile rheumatoid arthritis?

prescribed exercise to maintain mobility; splinting of affected joins; and teaching about medication management and side effects of drugs

What are two nursing priorities for a newborn with myelominingocle?

prevention of infection of the sac and monitoring for hydrocephalus (measure head circumference, check fontanel, assess neurologic functioning)

In developing a teaching plan for a 5 year old child w/ diabetes, which component of diabetic management should the nurse plan for the child to manage first?

process of glucose testing

A nurse is developing a postoperative plan of care for an infant who will undergo a pyloromyotomy for the treatment of hypertrophic pyloric stenosis. The nurse documents in the plan that the infant should be placed in which position in the postoperative period? a. supine with the head of the bed elevated b. prone with the head of the bed elevated c. flat on the nonoperative side d. flat on the operative side

prone with the head of the bed elevated

What are the priorities for a client with a Wilms tumor?

protect the child from injury to the encapsulated tumor. Prepare the family and child for surgery

"A 10-month-old infant is hospitalized for respiratory syncytial virus (RSV). Using knowledge of growth and development according to Erik Erikson and Jean Piaget, the nurse should do which of the following to meet the infant's developmental needs?" a. wash hands, wear a mask and keep the infant as quiet as possible b. follow the home feeding schedule and allow the infant to be held only when the parents visit c. restrain the infant continuously to prevent tubes from being dislodged d. provide a consistent routine, as well as touching, rocking, and cuddling throughout the hospitalization

provide a consistent routine, as well as touching, rocking, and cuddling throughout the hospitalization

describe the nursing care of a child with ketoacidosis

provide care for an unconscious child, administer regular insulin IV in normal saline, monitor blood gas values and maintain strict I/O

Preoperative nursing care for a child w/ Wilm's tumor should include which intervention?

put a sign on the bed reading, "DO NOT PALPATE ABDOMEN"

When evaluating the effectiveness of interventions to improve the nutritional status of an infant w/ gastro-esphageal reflux, which intervention is most important for the nurse to implement?

record weight daily The most definitive measure of improved nutrition is an infant is obtaining the child's daily weight

what are the two objectives in treating CHF?

reduce the workload of the heart increased cardiac output

A nurse is teaching the client taking medications by inhalation about the advantages of a newly prescribed spacer device. The nurse determines the need for further teaching if the client states that the spacer device: a. reduces the frequency of medication to only once per day b. reduces the chance of yeast infection because large drops arn't deposited on the oral tissues c. disperses medication more deeply and uniformly d. reduces the need to coordinate timing between pressing the inhaler and inspiration

reduces the frequency of medication to only once per day

describe why a barium enema is used to treat intussusception

reduces the telescoping of the intestine through hydrostatic pressure without surgical interventions

A 6 month old returns fr. surgery w/ elbow restraints in place. What nursing care should be included when caring for any restrained child?

remove restraitnts one at a time and provide range of motion exercises

"A nurse is caring for a child with intussusception. During care, the child passes a normal brown stool. The most appropriate nursing action is to:" a. report the passage of a normal brown stool to the physician b. prepare the child and parents for the possibility of surgery c. note the child's physical symptoms d. prepare the child for hydrostatic reduction

report the passage of a normal brown stool to the physician

To take the VS of a 4 month old child, which order provides the most accurate results?

respiratory rate, heart rate, then rectal temperature

List seven signs of respiratory distress in a pediatric client

restlessness tachycardia tachypnea diaphoresis flaring nostrils retractions grunting

A 3 yr old client w/ sickle cell anemia is admitted to the ER w/ abdominial pain. The nurse palapates an enlarged liver, and x ray reveals an enlarged spleen, and a CBC reveals anemia. These findings indicate which type of crisis?

sequestration this support a sequestration crisis where blood pools in the spleen, and is characterized by abdominal pain anemia

The community health nurse is providing a yearly summer educational session to parents in a local community. The topic of the session is prevention and treatment measures for poison ivy. The nurse instructs the parents that if the child comes in contact with poison ivy they should: a. immediately bring the child to the ER b. not be concerned if a rash is not noted on the skin c. shower the child immediately, lathering and rinsing the child several times d. apply calamine lotion immediately to the exposed skin areas

shower the child immediately, lathering and rinsing the child several times

What are the physical features of a child with down syndrome?

simian creases in palms, hypotonia, protruding tongue, and upward-outward slant of eyes

How is skeletal traction applied?

skeletal traction is maintained by pins or wires applied to the distal fragment of the fracture

Toddlers are at high risk for injuries because of their increasing curiosity, advancement in cognition, and improved motor skills. All these hazards are a concern for this age-group except: 1. burns 2. poisoning 3. sports injury 4. falls

sports injury- a toddler clearly would not be involved in team sports. :)

http://wps.prenhall.com/chet_ball_pedianurs_4/65/16689/4272478.cw/content/index.html

study also

What nursing interventions increase intracranial pressure?

suctioning and positioning, turning

A client is taking albuterol (Ventolin) by inhalation but cannot cough up secretions. The nurse teaches the client to do which of the following to best help clear the bronchial secretions? a. administer an extra dose before bedtime b. take in increased amounts of fluids every day c. get more exercise each day d. Use a dehumidifier in the home

take in increased amounts of fluids every day

A 17 yr old male student reports to the school clinic one morning ofr a scheduled health exam. He tells the nurse that he just finished football practice and is on his way to class. The nurse assess his VS: temp 100, pulse, 80, RR 20, and BP is 122/82. What is the best action for the nurse to take?

tell the student to proceed directly to his regulary scheduled class.

Which class of antiinfective drugs is contraindicated for use in children under 8 yrs of age?

tetracyclines

"A 12-year-old child is seen in the health care clinic. During the assessment, which finding would suggest to the nurse that the child is experiencing a disruption in the development of self-concept?" a. the child has a part-time babysitting job b. the child enjoys playing chess and mastering new skills with this game c. the child has many friends d. the child has an intimate relationship with a significant other

the child has an intimate relationship with a significant other

What instructions should a child with scoliosis receive about a skeletal brace?

the child should be instructed to wear brace 23 hours per day; wear T-shirt under brace; check skin for irritation; perform back and abdominal exercises; and modify clothing. The child should be encouraged to maintain normal activities as able

"A child with a tracheal obstruction is brought to the emergency room by emergency medical services. The child has aspirated a marble, and the foreign body is removed in the emergency room by direct laryngoscopy. After the procedure, the nurse informs the mother of the child that:" a. the child will need to be hospitalized for observation b. the child may go home with a prescription for antibiotics c. the child will need to return to the hospital for a chest x-ray in 1 week d. the child will require a bronchoscopy for follow up evaluation in 1 month

the child will need to be hospitalized for observation

A preschool-aged child who is hospitalized fy hypospadias repair is most strongly influenced by which behavior?

the preschoolers major stressor is concern for his body integrity.

describe the purpose of bronchodilators

to reverse bronchospasm

An infant brought to the emergency room is unresponsive and in respiratory distress. The nurse opens the infant's airway by which method? a. hyperextension b. jaw thrust c. tongue-jaw lift d. head tilt/ chin lift

tongue-jaw lift

The nurse assigning care for 5 yr old child w/ otitis media is concerned about the child's inceasing temperature over the past 24 hours. Which statement is accurate and should be considered when planning care for the remainder of the shift?

tympanic and oral temps are equally accurate

what are the signs and symptoms of congenital dislocated hip in infants?

unequal skin folds of the buttocks, ortolani sign, limited abduction of the affected hip and unequal leg lengths

what position does a child with epiglottitis assume?

upright sitting, with chin out and tongue protruding (tripod positioning)

Which action by the nurse is most helpful in communicating w/ a preschool aged child?

use a doll to play and communicate

To assess the effectiveness of an analgesic administered to a 4-yr old, what intervention is best for the nurse to implement?

use a happy-face/sad face pain scale.

The nurse is teaching a mother to give 4ml of a liquid antibiotic to a 10 month old infant. Which statement by the parent indicates a need for further teaching?

using a teaspoon will help me measure this correctly The prescribed medication is 4ml dosage and is measured w/ the most accuracy using a syringe, so if the parent uses teaspoon which is equiavelnt to 5ml, further teaching is indicated

A nurse is preparing a plan of care for a child being admitted to the hospital with a diagnosis of congestive heart failure (CHF). The nurse avoids including which of the following in the plan? a. limiting the time the child is allowed to bottle-feed b. elevating the head of the bed c. waking the child for feeding to ensure adequate nutrition d. providing oxygen during stressful periods

waking the child for feeding to ensure adequate nutrition

list normal findings in a neurovascular assessment

warm extremity, brisk capillary refill, free movement, normal sensation of the affected extremity and equal pulses

The nurse is providing instructions to the parents of a child with a hernia regarding measures that will aid in reducing the hernia. The nurse determines that the parents understand these measures if they state which of the following? a. we will be sure to give our child a fleet enema every day to prevent constipation b. we will make sure that our child participates in physical activity every day c. we will provide comfort measures to reduce any crying periods by our child d. we will encourage our child to cough every few hours on a daily basis

we will provide comfort measures to reduce any crying periods by our child

A client with diplopia has been given an eye patch to promote better vision and prevent injury. The nurse teaches the client to do which of the following as part of correct use of this item? a. wear the patch continuously, alternating eyes each day b. wear the patch continuously, alternating eyes each week c. use the patch only when vision is exceptionally blurry d. put the patch on for an hour, and then take it off for an hour

wear the patch continuously, alternating eyes each day

What measurements reflect present nutritional status?

weight, skin-fold thickness, and arm circumference

"A client taking albuterol (Ventolin) experiences a severe episode of wheezing, which the nurse interprets as bronchospasm. A telephone call is made to the physician's office to report the occurrence. The nurse does which of the following while waiting for the physician to return the call?" a. administer the next dose of albuterol as scheduled b. withholds the next dose of albuterol c. administer a double dose of albuterol d. administers half the dose of albuterol

withholds the next dose of albuterol


Set pelajaran terkait

Principles of Business Final Study Set

View Set

Chapter 7: The Skeletal System/Chapter 8: Joints

View Set

Slope-Intercept Form of a Line Writing an Equation to Represent a Real-World Problem

View Set

Med surg 205 inflammation and wound healing

View Set

Linear Algebra Lecture Questions

View Set

Quiz 4: Hand muscles origin, insertions, actions, innervation

View Set

Origin of Modern Astronomy & Touring Our Solar Systems: Chapter 21, 22

View Set